Board Review Questions Emergency

37 100. A 30-year-old was involved in a rapid deceleration motor vehicle crash at street speed. He was gripping the stee

Views 124 Downloads 2 File size 2MB

Report DMCA / Copyright

DOWNLOAD FILE

Recommend stories

Citation preview

37 100. A 30-year-old was involved in a rapid deceleration motor vehicle crash at street speed. He was gripping the steering wheel tightly, and complains of pain at the MCP joint of the right thumb. On exam there is tenderness and mild swelling of the joint on the ulnar side. X-rays are negative. Which of the following is correct regarding the evaluation and treatment of this injury? a. If the ulnar collateral ligament demonstrates more than 35o of laxity when compared to the uninjured side, place a thumb spica splint and refer to orthopedics for probable surgical management. b. If the radial collateral ligament demonstrates any laxity, refer to orthopedics for surgical repair. c. All gamekeepers’ thumbs need to be repaired surgically. d. The absence of a fracture indicates that no splinting will be necessary. e. If the ulnar collateral ligament demonstrates less than 20o laxity, apply a universal wrist splint, and refer for surgical repair. 101. This x-ray shows a: a. dorsal dislocation of the lunate. b. volar dislocation of the lunate. c. dorsal perilunate dislocation. d. volar perilunate dislocation. e. scapholunate dissociation.

Rosen 5th, Figure 44-11

102. Elbow dislocations are associated most commonly with concomitant injuries to the: a. ulnar nerve and radial artery. b. ulnar nerve and brachial artery. c. radial nerve and ulnar artery. d. radial nerve and brachial artery. e. median nerve and radial artery. 103. This child fell while rollerblading and has a tender elbow. This x-ray (figure 169-15) is most consistent with: a. nursemaid’s elbow. b. radial head fracture. c. proximal ulnar fracture. d. elbow dislocation. e. supracondylar fracture

Rosen’s 5th, Figure 169-15

104. Monteggia’s injury is a : a. fracture of the ulnar shaft with radial head dislocation. b. fracture of the ulnar shaft with distal radioulnar dislocation. c. fracture of both the radius and ulna. d. distal radial fracture with radioulnar dislocation. e. distal radial fracture with radial head dislocation 105. This x-ray (Figure 44-24) demonstrates a: a. Colles fracture. b. Monteggia fracture. c. Galleazzi fracture. d. Bennett fracture. e. Barton fracture.

Rosen 5th, Figure 44-24

38

100. a th

Rosen 5 , Chapter 43, p. 520

101. c Rosen 5th, Chapter 44, p. 541

102. b Rosen 5th, Chapter 45, p. 572

103. e th

Rosen 5 , Chapter 45, p. 564

104. a Rosen 5th, Chapter 44, p. 551

105. c Rosen 5th, Chapter 44, p. 552

Injury to the ulnar collateral ligament (UCL) was first described as an occupational hazard of Scottish gamekeepers who damaged their thumbs by repeatedly twisting the necks of hares. Skiing is now the most common cause of both acute and chronic injury to the UCL. UCL rupture occurs ten times more often than radial collateral ligament injury. The mechanism of injury is forced radial deviation (abduction), and the subsequent tear usually occurs at the insertion into the proximal phalanx. Complete and partial ruptures can usually be differentiated by clinical examination. Valgus stress testing of the ulnar collateral ligaments is required and should be performed in full extension and in 30% flexion to avoid the stabilizing effect of the volar plate. More than 35% of joint laxity or 15% more laxity than is present in the uninjured thumb is consistent with complete UCL rupture. Acute partial ruptures of the UCL can be effectively treated by a 4-week period of immobilization in a thumb spica cast, and full recovery is the rule. Complete ligament tear requires surgical repair because a high number have associated soft tissue interposition from the adductor aponeurosis (Stener’s lesion) with limited predicted healing potential. A perilunate dislocation is best seen on the lateral view of the wrist. Although the lunate remains in position relative to the distal radius, the capitate is dorsally dislocated. The PA view shows overlap of the distal and proximal carpal rows and may also demonstrate an associated scaphoid fracture or subluxation. In patients with elbow dislocations, neurovascular complications occur in 8 to 21%, the most frequent being injury to the ulnar nerve. Vascular complications occur in 5 to 13% of elbow dislocations, with brachial artery injury the most common. Supracondylar injuries are extra-articular fractures which occur most commonly in children; 95% are displaced posteriorly as a result of an extension force. The patient will have significant swelling and tenderness at the elbow. Radiographs may reveal a fat-pad sign in undisplaced fractures due to visualization of fat from the olecranon fossa (posterior fat pad) as it is displaced by the hemarthrosis. In some undisplaced fractures, the fracture line may not be seen, with the fat-pad sign being the only evidence of injury. Treatment should be initiated as though a fracture were identified, with splint immobilization and orthopedic consultation. Fracture of the ulnar shaft with a radial head dislocation is often referred to as Monteggia fracture-dislocation. It is typically a diaphyseal fracture in the proximal third of the ulna with an anterior dislocation of the radial head (60% of cases). Anterolateral and posterolateral dislocation of the radial head or a metaphyseal ulna fracture are other possibilities. As a rule, the radial head normally points to the capitellum in all radiographic views of the elbow. In a Monteggia fracture, the apex of the ulna fracture points in the direction of the radial head dislocation. A Galeazzi fracture involves the junction of the middle and distal thirds of the radius, with an associated dislocation or subluxation of the distal radial-ulnar joint (DRUJ). This rare injury accounts for only 7% of all fractures of the forearm. It occurs as a result of a fall on the outstretched hand, with the wrist in extension and the forearm forcibly pronated. The radial fracture causes obvious swelling and deformity on the radial side of the forearm, and pain is localized over the fracture site. In addition, the DRUJ is swollen and painful on palpation, and the ulnar head appears prominent when compared with the unaffected wrist.

39 106. A 16 year old boy complains of pain and swelling in his right hand after hitting a wall with a closed fist. You know that: a. the neck of the metacarpal is the most frequently fractured segment. b. metacarpal fractures of the long and index finger do not need to be reduced if there is less than 30o of angulation. c. metacarpal fractures of the ring and small fingers do not need to be reduced if there is less than 20o of angulation. d. most metacarpal fractures angle in a volar direction. e. a boxer’s fracture refers to metacarpal neck fractures of the long and sometimes ring fingers. 107. A 5 year old boy falls and fractures his radius. a. Salter-Harris Type I fractures are the most common type b. The picture shows a Salter-Harris Type II fracture. c. Salter-Harris Type III fractures involve the epiphyseal plate and have a widened space between epiphysis and metaphysis. d. Salter-Harris Type IV fractures are difficult to diagnose using standard radiographs. e. Salter-Harris Type I, II, and III injuries have poor prognosis despite proper reduction and immobilization. 108. A 29 year-old man is tackled while playing football and suffers what appears to be a shoulder dislocation. You know that: a. the most common direction of dislocation is posterior. b. anterior-posterior (AP) and lateral radiographs are the best views to confirm the diagnosis. c. after reduction he should be placed in a shoulder immobilizer for 2 months. d. infraglenoid dislocations are associated with seizures. e. inadequate muscle relaxation is the most common reason for failed reduction. 109. A 43-year-old softball player pulled up lame stretching a single into a double. You suspect a ruptured Achilles tendon, since you know: a. when the soleus and gastrocnemius muscles contract, the Achilles tendon pulls up the calcaneus, dorsiflexing the foot. b. rupture often occurs in professional sports settings, especially in over-conditioned athletes. c. the patient hears a popping sound and then has difficulty walking. d. the Thompson-Doherty test is performed by squeezing the midportion of the calf while the patient is lying in the prone position; an intact Achilles tendon is demonstrated by dorsiflexion of the foot. e. unbearable pain is a cardinal symptom. 110. A 23-year-old hairdresser complains of two days of increasing pain and swelling in her right index finger pad. She has no fever. Her finger pad is tense, red, warm, and very tender. Her nail and paronychial region are not affected. You should: a. place her on amoxicillin / clavulanate (Augmentin®), elevation, and warm soaks. b. perform a through-and-through incision of the pad volar to the neurovascular bundle, then amoxicillin / clavulanate (Augmentin®), elevation, and warm soaks. c. perform a fish mouth incision and refer to a hand surgeon. d. perform a unilateral longitudinal incision on the ulnar side dorsal to the neurovascular bundle, amoxicillin / clavulanate (Augmentin®), elevation, warm soaks, pain medicine, recheck in two days. e. remove the nail, incise the eponychium, hot soaks, pain meds, and recheck in two days.

40

106. d th

Rosen 5 , Chapter 43, p. 513

107. c Rosen 5th, Chapter 169, p. 2373

108. e th

Rosen 5 , Chapter 46, p. 592-594

109. c Rosen 5th, Chapter 51 p.716

110. d Rosen 5th, Chapter 43, p. 529-530

Fractures of the metacarpal neck are among the most common fractures in the hand. They are usually due to a direct impaction force. A fracture of the fifth metacarpal neck is often referred to as a boxer’s fracture. These fractures are usually unstable with volar angulation. Angulation of less than 20° in the fourth and 40° in the fifth metacarpal will not result in functional impairment. In the second and third metacarpal, angulation of 50%. 115. Ankle sprains: a. are common in young children and toddlers. b. most commonly involve the anterior talofibular ligament. c. most commonly occur on the medial side of the ankle. d. require radiography to diagnosis a third-degree injury. e. involving isolated deltoid ligament injuries are very common.

42

111. e th

Rosen 5 , Chapter 44, p. 538

112. c Rosen 5th, Chapter 47, p. 617

113. d Rosen 5th, Chapter 48, p. 640

114. b Rosen 5th, Chapter 49, pages listed in answer

115. b Rosen 5th, Chapter 51 p.714

A stage I injury, or scapholunate dissociation, results in a characteristic widening of the scapholunate joint on the PA view, which has been called the Terry-Thomas sign, after the British comedian with a gap between his front teeth. When a scapholunate ligament injury is suspected clinically, additional stress views should be obtained. Views taken with a clenched fist and ulnar deviation (the clenched fist AP view) will accentuate widening of the scapholunate joint. The most important test for back pain is the straight leg raise. This is done with the patient supine and the legs extended, or with both legs slightly flexed at the hips and knees. The symptomatic leg is passively raised, with the knee fully extended. Pain caused by a disk herniation in L4-L5-S1 usually results in increased pain in the leg at anywhere from 30 to 70o of elevation, as the affected nerve root is stretched. It is important to note that back pain, probably secondary to stretching of the paravertebral muscles as the leg is elevated, indicates absolutely nothing about radiculopathy. A straight leg raise test is positive only if lifting the affected leg reproduces or worsens the pain in that leg. The pain of radiculopathy is usually worse in the leg than in the back and almost always radiates past the knee. Ninety-five percent of disk herniations occur at L4–L5 or L5–S1. Without complaint of sciatica, or pain below the knee in a dermatomal distribution, the chance of a herniated nucleus pulposus is 1 in 1000. When attempts to stabilize the patient hemodynamically with crystalloid infusion and transfusion of blood products are unsuccessful, application of a pneumatic anti-shock garment should be discussed with orthopedic and general surgical consultants. Although controversial, PASG can temporarily splint the fracture and tamponade pelvic bleeding until definitive stabilization and angiography with embolization can be performed.. The Allis maneuver (Figure 49-27) should be attempted in order to reduce a [posterior hip dislocation as quickly as possible. The patient is placed supine  the knee is flexed and steady traction is applied in line with the deformity – the hip is brought slowly to 90o of flexion  after reduction, the hip is extended while maintaining traction (p. 664). A patient with a femoral neck fracture presents with leg externally rotated and shortened. A patient with a posterior hip dislocation typically holds the hip flexed, adducted, and internally rotated (p. 662). Sciatic palsies, especially the peroneal nerve branch, occur in 10% of patients with posterior hip dislocation (p. 662). Acetabular fractures are common (p. 662). Avascular necrosis is reported to occur in 1% to 17% of patients with this injury (p. 665). Most ankle sprains occur from extreme inversion and plantarflexion. Approximately two thirds of ankle sprains are isolated anterior talofibular ligament injuries, whereas 20% involve both anterior talofibular and calcaneofibular ligament injuries. Isolated injury of the deltoid ligament occurs in fewer than 5% of ankle sprains. A grade III sprain involves a complete tear of the ligament with marked joint instability and severe edema and ecchymosis, and is a clinical diagnosis.

43 116. A 75-year-old woman fell at a local flower shop and complains of severe left hip pain. Her left leg is shortened and externally rotated. She has strong pedal pulses and sensation is intact. Thorough head-to-toe examination reveals no other injuries. Her left hip x-ray is shown (Figure 49-17), so you arrange for the patient’s admission to the orthopedic doctor on call. The patient’s daughter, a pediatrician, wants to know about potential complications from this injury. You tell her that, just as in children, the most common complication is: a. avascular necrosis. Rosen 5th, b. hemorrhage. Figure 49-17 c. osteomyelitis. d. fat embolism. e. femoral nerve injury. 117. A 24 year-old basketball player severely twisted his left knee several hours ago and is unable to bear weight. There is a large joint effusion. While examining his knee, you see that valgus stress at 30o flexion causes more than a centimeter of “give” without a firm endpoint. The knee is stable in extension with valgus stress. This suggests: a. an isolated medial collateral ligament disruption. b. an isolated lateral collateral ligament disruption. c. a medial collateral ligament tear with probable involvement of the cruciates and posterior capsule. d. a lateral collateral ligament rupture with posterior cruciate tear. e. probable anterior cruciate rupture with meniscal tear. 118. A 32-year-old man swung at a pitch while playing softball, then collapsed due to severe knee pain. His injured leg looks like this. Your treatment should be: a. immobilization, x-ray, pain medicine, and examination for probable peroneal nerve injury. b. needle arthrocentesis of the prepatellar bursa, then ice and elevation. c. orthopedic consult for repair of the patellar tendon. d. gentle lateral pressure on the patella with the knee in extension. e. examine for possible popliteal artery disruption. 119. A 35-year-old male skier fell and twisted his knee. Within five minutes, he developed severe pain and a large joint effusion. When testing his knee stability, you know that: a. the anterior drawer sign is highly accurate in diagnosing acute ACL injuries. b. abduction, flexion, and internal rotation of the femur on the tibia produce injuries to the lateral side of the knee. c. adduction, flexion, and external rotation of the femur on the tibia produce injuries to the medial side of the knee. d. lateral knee injuries are far more common than medial knee injuries. e. a negative Lachman’s test rules out ACL injury with >90% accuracy. 120. Metabolic acidosis without an anion gap is caused by: a. cyanide poisoning. b. renal tubular acidosis. c. lactic acidosis. d. ketoacidosis. e. salicylate ingestion.

44

116. a th

Rosen 5 , Chapter 49, p. 656

117. a Rosen 5th, Chapter 50, p. 678

118. d Rosen 5th, Chapter 50 p.688

119. e Rosen 5th, Chapter 50 p.677

In a patient with femoral neck fracture, avascular necrosis is the most common complication, despite optimal treatment, because of the complex arterial anatomy. The mortality rate during the first year after a femoral neck fracture is 14%, compared with 9% for the control population. Deep infection, in the form of osteomyelitis or septic arthritis, is more common with femoral neck fractures because the fracture line extends into the joint. Pulmonary embolism is the leading cause of death at 7 days postfracture in all orthopedic patients. Abduction or valgus pressure applied to a knee in approximately 30° of flexion determines the integrity of the medial capsular and ligamentous structures. If there is a demonstrated laxity of greater than 1 cm without a firm end-point as compared to the other knee, there is a complete rupture of the medial collateral ligament. If there is laxity with a firm end-point or a laxity of less than 1 cm, an incomplete or partial tear is present. If there is no demonstrated instability but there is pain, the patient has suffered a strain. The patient who is unstable with the valgus test performed with 30° of flexion should be brought into full extension, if possible, and similar maneuvers carried out. Medial instability in full extension indicates a severe lesion involving the cruciate ligaments and posterior capsule along with the medial ligaments. Patellar dislocation may occur from an athletic injury caused by valgus stress combined with flexion and external rotation. Patients may complain of the knee giving out accompanied by pain and swelling. Inability to bear weight or to flex the knee is a common complaint. Examination shows an anterior defect with the patella deviated laterally. Closed reduction is accomplished through force or pressure directed anteromedially on the lateral patellar margin while simultaneously gently extending the leg. A tear in the anterior cruciate ligament or intra-articular fracture usually results in an immediate hemarthrosis and inability to continue activity. A torn meniscus may cause an acutely locked knee but more commonly has delayed onset of swelling over 12 to 24 hours and intermittent locking associated with joint line pain. Lachman’s test is currently the single best clinical test for determining the integrity of the ACL and one of the only reliably performed tests in a patient with an acute hemarthrosis. Accuracy in diagnosing ACL injury increases from 70% to 99% using Lachman’s test rather than the anterior drawer test. The anterior drawer test is not reliable and is of little value in diagnosing acute ACL injuries.

120. b Elevated Anion Gap th

Rosen 5 , Chapter 118 p.1719

(“CAT MUD PILES”) Carbon monoxide / Cyanide exposure Acute alcohol intoxication / Alcoholic ketoacidosis Toluene exposure Methanol intoxication Uremia Diabetic ketoacidosis Paraldehyde ingestion Isoniazid (INH) / Iron intoxication Lactic acidosis Ethylene glycol intoxication Salicylate intoxication

Normal Anion Gap

Gastrointestinal loss of HCO3– Diarrhea Enterostomy Ureteroenterostomy Renal loss of HCO3– Renal tubular acidosis Acetazolamide Hyperalimentation

45 121. Which statement is correct concerning the entity shown in this x-ray? a. This is a common foot injury, usually caused by inversion. b. Associated fractures are quite common, and usually involve the base of the 2nd metatarsal. c. This is a pseudo-pseudo-Jones fracture. d. Rest, ice, elevation, and ambulation as tolerated give the best longterm results. e. You can easily reduce this injury with gentle axial traction and plantar flexion.

Rosen 5th, Figure 51-24

122. A 35-year-old man accidentally chopped off his left index finger at the PIP joint about six hours ago while cutting firewood at a campsite in a local state park. He stuck the amputated finger in his pocket, hiked out of the woods, and drove to your Emergency Department. You should: a. clean the amputated finger with Betadine® and place it in the refrigerator. b. clean the amputated finger with hydrogen peroxide, wrapped in saline gauze, and place it on ice. c. irrigate the amputated finger with saline, wrap it in sterile dry gauze and place it in a plastic bag, which should be immersed in ice. d. Irrigate the amputated finger with saline, wrap it in saline-moistened gauze and place it in a plastic bag, which should be immersed in ice-water slurry. e. There is no need to care for the amputated extremity as the ischemic time is too long and no reimplantation will be possible 123. A 20-year-old college student has redness, tenderness, and swelling on the plantar surface of his foot. Two days ago, while wearing sneakers, he stepped on a nail. You should prescribe: a. cephalexin. b. amoxicillin/clavulanate. c. ciprofloxacin. d. tetracycline. e. erythromycin. 124. Antibiotics are clearly indicated for: a. all dog bites. b. all cat bites. c. all human bites. d. all foot puncture wounds. e. all intraoral lacerations. 125. The acute life threats in adrenal inufficiency are: a. hypertension and hyperglycemia. b. hyperthermia and hypoglycemia. c. hypothermia and hyperglycemia. d. hypotension and hypoglycemia. e. hyperthermia and hypertension. 126. A patient being treated for diabetic ketoacidosis develops weakness and paresthesias. The most likely metabolic abnormality is: a. acidosis. b. hypokalemia. c. hypophosphatemia. d. hypocalcemia. e. hypoglycemia.

46

121. b th

Rosen 5 , Chapter 51, p. 726

122. d Rosen 5th, Chapter 43, p. 527

123. c Rosen 5th, Chapter 52, p. 748

124. b Rosen 5th, Chapter 52, p. 748

125. e Rosen 5th, Chapter 122, p. 1781

126. b th

Rosen 5 , Chapter 120, p. 1753-1754

The six-bone tarsometatarsal complex is known as the Lisfranc joint. Up to 20% of fractures at this joint are missed in the ED. The force required and the mechanisms of injury are varied and can range from a seemingly minor rotational force to severe axial load as seen in an automobile accident. The great majority of injuries to the Lisfranc joint are associated with fractures, usually of the metatarsals, the cuboid, or the cuneiforms. A fracture of the base of the second metatarsal is pathognomonic of a disruption of the ligamentous complex. Injuries to the Lisfranc joint frequently require open reduction and fixation or percutaneous placement of Kirschner wires and nonweight-bearing for several weeks. The time that an amputated part can survive before reimplantation has not been determined. As a general rule, the more proximal the amputation, the less ischemic time the amputated part can tolerate. Warm ischemia may be tolerated for 6 to 8 hours, but there are reports of successful replantation of digits after 30 hours of warm ischemia. The amputated part requires minimal handling and should be cooled as soon as possible. After wrapping in saline-moistened gauze, the part is sealed in a dry plastic bag and placed in ice water. Ice should not come in direct contact with the tissue because this can cause local damage. Local antiseptics, especially hydrogen peroxide or alcohol, should not be used because they may damage viable tissues. The infection rate for foot puncture wounds has been reported to be as high as 15%. Most infections are due to gram-positive organisms, with Staphylococcus aureus predominating, followed by other staphylococcal and streptococcal species. Pseudomonas aeruginosa is the most frequent pathogen isolated post-puncture wound osteomyelitis, particularly when foreign-body penetration occurs through the sole of an athletic shoe. Because this organism is not detected in new shoes, it has been postulated that the foam rubber material becomes colonized in the warm, humid summer months. The only oral agents consistently effective against Pseudomonas are the fluoroquinolones, including ciprofloxacin and levofloxacin. Antibiotic prophylaxis is required for all patients with cat bites. Antibiotic prophylaxis for dog bites is more controversial, and seven of eight randomized trails of dog bite wounds show no benefits with antibiotics. While human bites of the hand are at a high risk for infection, human bites of other locations appear to have no worse infection rate than other wounds. No data suggest a benefit from prophylactic antibiotics in foot puncture wounds. Little data suggest a clear indication for prophylactic antibiotics in patients with intraoral lacerations, but through-and-through wounds have an infection rate twice as high as simple mucosal injuries. The acute life threats in adrenal insufficiency are hypotension and hypoglycemia. Hypotension responds well to glucocorticoid replacement with IV hydration, and hypoglycemia to IV administration of D5W. Potassium replacement is invariably needed in DKA. The initial potassium level is often normal or high despite a large deficit because of severe acidosis. Potassium levels often plummet with correction of acidosis and administration of insulin. Potassium should be administered with the fluids while the laboratory value is in the upper half of the normal range.

47 127. According to his father, this 11-year-old boy has complained about a vague hip, thigh, and knee pain for the past 2 weeks. He recalls no injury. There has been no fever. The child walks with a limp and winces when his right foot hits the ground. His x-ray demonstrates: a. osteomalacia femoralis. b. Osgood-Schlatter disease. c. Legg-Calvé-Perthes disease. Rosen 5th, d. septic synovitis. Figure 49-34 e. slipped capital femoral epiphysis. 128. In a patient with primary adrenal insufficiency you would expect to see: a. hypernatremia. b. hypokalemia. c. hypoglycemia. d. azotemia. e. acidosis. 129. The most common cause of a markedly elevated serum potassium is: a. laboratory error. b. renal failure. c. Addisonian crisis. d. rhabdomyolysis. e. acidosis. 130. Hirschsprung's disease: a. is typically diagnosed in infancy. b. almost always is associated with enterocolitis. c. is more frequent in females. d. can be diagnosed by ultrasound. e. can best be managed pharmacologically. 131. You deliver a child at 36-weeks gestation and intestines are protruding from a defect in the abdominal wall. On inspection, you note that the intestines lack a peritoneal covering. a. A gastroschisis is a defect in the umbilical ring that allows the intestines to protrude out of the abdominal cavity in a sac. b. An omphalocele is a defect in the abdominal wall that allows the evisceration of abdominal structures without a sac being present. c. Gastroschisis and omphalocele are invariably fatal within hours. d. You are twice as likely to encounter gastroschisis you are to see omphalocele. e. Emergency department treatment gastroschisis involves gastric decompression; this same treatment is contraindicated in omphalocele. 132. A 13-year-old male complains of acute, severe pain and swelling in his left testicle. He says that the pain woke him from sleep. He had something silimar a week or two ago, but it got better spontaneously. Suspecting torsion testicle, you know that: a. absence of the cremasteric reflex is not a useful physical exam finding. b. if this patient's symptoms have only been present for 1 hour, Doppler imaging of the testicles can be obtained before obtaining surgical consultation. c. relief of this pain with elevation of the scrotum indicates that epididymitis is the diagnosis. d. torsion of the right testicle is more common due to a longer spermatic cord. e. testicular pain may be constant or intermittent, but it is not positional.

48

127. e th

Rosen 5 , Chapter 169 p.2387

128. c Rosen 5th, Chapter 122 p.1781

129. a Rosen 5th, Chapter 119, p. 1730

130. a Rosen 5th, Chapter 165, p. 2305-2306

131. d Rosen 5th, Chapter 9, p. 103

132. e Rosen 5th, Chapter 94 p.1423

Children with a stable slipped capital femoral epiphysis have symptoms of intermittent limp and pain for several weeks to months. Stable slips make up approximately 90% of all cases. The pain may be localized to the hip but more commonly is poorly localized to the thigh, groin, or knee. On AP radiographs, signs of slippage include Klein’s line drawn along the superior margin of the femoral neck. With a normal hip, the line intersects or falls within the epiphysis, whereas in a hip with a slipped epiphysis, the line does not come in contact with the epiphysis. Two thirds of patients with adrenal failure have associated hypoglycemia. The symptoms are characteristic of hypoglycemia: perspiration, tachycardia, weakness, nausea, vomiting, headache, convulsions, and coma. The pathophysiology is decreased gluconeogenesis and increased peripheral glucose use secondary to lipolysis. Electrolyte abnormalities are common. Hyponatremia is present in 88% of cases, hyperkalemia in 64%, either hyponatremia or hyperkalemia in 92%, and hypercalcemia in 6% to 33%. When faced with a report of a high serum potassium level, the Emergency Physician should first consider the possibility of laboratory error, the most common cause of hyperkalemia. Hemolysis during phlebotomy, as may occur when blood is obtained with a small needle or sampled in a high-vacuum tube, releases potassium into the sample and causes a spuriously high potassium level to be measured. Hirschsprung’s disease accounts for 20% of partial intestinal obstruction early in infancy. It is 4 to 5 times more common in boys and can be associated with Down syndrome or a variety of other anomalies of the GI, GU, or neurologic systems. Neonates often present in the nursery with a failure to pass meconium. Diagnosis is suspected on barium enema, and confirmed by biopsy. Treatment in surgical. Gastroschisis is a defect in the abdominal wall that allows the antenatal evisceration of abdominal structures without a sac being present. Omphalocele is a defect in the umbilical ring that allows the intestines to protrude out of the abdominal cavity in a sac. Gastroschisis occurs twice as frequently as omphalocele. ED treatment involved gastric decompression and placing an occlusive plastic covering to prevent heat and water loss. Manual detorsion is never curative but should be attempted in most cases while awaiting surgical treatment. Prehn's sign, or relief of pain with elevation of the scrotum, was historically thought to be associated with epididymitis, but this association is false and should not be used to distinguish torsion from epididymitis. The cremasteric reflex is almost always absent in testicular torsion. The salvage rate in testicular torsion remains high if pain has lasted less than 6 hours, but a urology consult should be obtained as soon as the diagnosis seems likely clinically.

49 133. Concerning intraosseous lines: a. Fat embolism is a common complication, especially in adult patients. b. In adults, the tibia is technically easier than in children because of relative proximity to the surface. c. The preferred insertion site in the adult femur is the lateral surface of the trochanter. d. Infection rates approach 20% in both adults and children. e. Any drug which can be given intravenously can also be delivered through an intra-osseous line. 134. A 3-month-old girl is irritable, feeds poorly, and breaks into a profuse sweat whenever she takes the nipple. She is tachypneic and pale. Her heart rate is 280 / minute. Chest x-ray shows a large heart with alveolar infiltrates. A rhythm strip is shown. You know that: a. jugular venous distension and peripheral edema are commonly found in infants with this condition. b. volume replacement is the most essential treatment c. heart rate is the least malleable of the cardiovascular parameters. d. cardioversion with 0.01 watt-sec/kg is indicated for profound shock e. a stable patient can be treated with vagal maneuvers 135. A 16-month-old child is brought to the emergency department immobilized in cervical spine precautions. The child was an unrestrained passenger in a moderate-speed rapid-deceleration motor vehicle collision. You are concerned about possible neck injury, keeping in mind that: a. pseudosubluxation of C3 on C4 is common in children. b. the small neck muscles make fractures more common than ligamentous injury. c. the predental space should not exceed 4 to 5 mm in children younger than 10. d. anatomic features of the cervical spine approach adult patterns at around 12 years of age. e. if the child was ambulatory at the scene, spinal precautions are unnecessary. 136. A 2-year-old child is brought to the emergency department because of fever and pulling at his left ear. Otitis media is best confirmed by: a. decreased mobility of his tympanic membrane and loss of normal landmarks. b. hyperemia of his tympanic membrane in the absence of crying. c. retraction of his tympanic membrane accompanied by upper respiratory infection symptoms. d. loss of the tympanic light reflex. e. the presence of otalgia and upper respiratory infection symptoms. 137. An 18-month-old boy has fever and poor feeding. His ears look normal, but he has an exudative pharyngitis and enlarged cervical lymph nodes. A rapid antigen detection test for Group A betahemolytic streptococcus (GABHS) is negative. a. Rapid streptococcal tests are highly sensitive, so it is highly unlikely that this child has acute GAS pharyngitis. b. While GABHS is unlikely at this age, detection is important because of the significant risk of acute rheumatic fever in untreated patients. c. If the test had been positive, appropriate treatment would include a dose of intramuscular procaine penicillin. d. If the test had been positive, early institution of appropriate antibiotics would likely have shortened the clinical course of the illness e. Infectious mononucleosis is not a consideration because of the child’s age.

50

133. e th

Rosen 5 , Chapter 8, p. 84

134. e Rosen 5th, Chapter 164 p. 2290-2292

135. c Rosen 5th, Chapter 32, p. 274-276

136. a Rosen 5th, Chapter 67, p. 928-931

137. d Rosen 5th, Chapter 70, p. 969-972

For pediatric patients up to 5 years of age, the tibia is the preferred site. In adults, the most commonly used site is the medial malleolus. Although complications such as fat and marrow emboli to the lungs have been reported, recent data suggest that the risk is not increased by the use of IO access for emergent fluid and drug administration. The tibia is technically more difficult in adults than in children because the adult bone is thicker and the needle tends to slip off. The incidence of infection, including both cellulitis and osteomyelitis, is less than 1%, and the potential for infection can be minimized by limiting the duration of intraosseous infusion and avoiding hypertonic solutions. The predominant symptoms of congenital heart disease include poor feeding, excessive diaphoresis, irritability or lethargy with feeding, weak cry and, in severe cases, grunting and nasal flaring. Tachypnea is a cardinal sign. Since feeding is the infant’s primary form of exertion, dyspnea and sweating during feeding can often be elicited in the history. Peripheral edema, jugular venous distention, and rales are unusual and late signs in infants. Heart rate is the most malleable of the cardiac physiologic parameters. The common pediatric dysrhythmia is paroxysmal supraventricular tachycardia. Initial management of unstable patients with narrow complex tachycardia consists of immediate synchronized cardioversion at 0.5 J/kg with increases in power output to 2 J/kg as needed. In the stable patient, vagal maneuvers are the intervention of choice. If vagal maneuvers are not successful in the stable patient, IV adenosine (0.1 mg/kg – maximum first dose 6mg) followed by (0.2mg/kg – maximum 12mg) can be used. On a lateral cervical spine view the distance between the anterior aspect of the odontoid process and the posterior aspect of the anterior ring of C1, the so-called predental space, should not exceed 5 mm in a child. Pseudosubluxation of C2 on C3 occurs in approximately 40% of children up to adolescence. Anatomic features of the cervical spine approach adult patterns between the ages of 8 and 10 years. Compared to adults, the child has relatively underdeveloped neck musculature and a head that is disproportionately large and heavy compared to the body, leading to fewer fractures and more ligamentous injuries. The most significant sign of acute otitis media (AOM) is the loss of or decrease in mobility of the tympanic membrane. The light reflex is of no diagnostic value. The normal eardrum is translucent and pearly gray but may become reddened with crying. The TM of AOM is usually opaque, hyperemic, and sometimes bulging, and bony landmarks (long and short process of the malleus) are not easily discernible. Early antibiotic treatment of streptococcal pharyngitis leads to a 13% earlier resolution of symptoms and shortens the course of illness by about 1 day. GABHS is primarily a disease of children 5 to 15 years old and occurs in winter and early spring. It is responsible for less than 15% of pharyngitis in patients older than 15 years of age and is rare in patients less than 3 years old. Rapid strep tests have reported specificity of 70% to 100% (with most being >95%) and sensitivity of 31% to 100% (with most being 60% to 95%). The incidence of rheumatic fever parallels that of GABHS, with the peak incidence in children 5 to 15 years old, less common in adults, and rare in children less than 3 years of age. Monospot tests are positive in about 30% of children 0 to 20 months with mononucleosis.

51 138. A 3-day-old baby presents to the emergency department with purulent conjunctivitis. His mother has a history of syphilis treated prior to this pregnancy. She had no prenatal care, but delivered in the hospital. The infant received topical erythromycin prophylaxis in the nursery. The most likely cause of this child’s illness is: a. syphilis. b. Neisseria gonorrhoeae. c. group B streptococcus. d. Herpes simplex. e. Chlamydia trachomatis. 139. A 2-year-old patient has screamed inconsolably for four hours and is brought to the ED by his frazzled parents. He is afebrile and non-toxic in appearance; initial exam shows nothing but mild left conjunctival erythema without discharge. Your next step should be: a. lumbar puncture. b. Schiotz tonometry. c. fluorescein staining of the cornea. d. urinalysis. e. stool for occult blood. 140. Parents bring a 6-week-old infant to the ED because of 3 days of vomiting, occurring immediately after eager feedings. The baby is fussy, but alert. His mucous membranes are tacky. When you feed him, he eagerly takes the bottle, but then promptly vomits. Except for the dehydration, you find nothing else on the exam. This child will require: a. surgical intervention. b. intravenous hydration and observation. c. prompt parenteral antibiotic therapy. d. discharge with BRAT diet and parental reassurance. e. change to a soybean-based formula. 141. Cyanosis frequently occurs with: a. ventricular septal defect. b. atrial septal defect. c. patent ductus arteriosus. d. tetralogy of Fallot. e. hypertrophic cardiomyopathy. 142. Regarding ear infections: a. The tympanic membrane should always appear normal in acute otitis externa. b. Normal tympanic membrane mobility rules out acute otitis media. c. Children with acute otitis media who are compliant with appropriate antibiotic regimens do not require any follow-up. d. An intense worsening of the pain usually indicates sudden tympanic membrane perforation. e. Topical antibiotic therapy for otitis externa is ineffective unless the canal is free of debris and cerumen.

52

138. e th

Rosen 5 , Chapter 66, p. 917

139. c Rosen 5th, Chapter 66

140. a th

Rosen 5 , Chapter 165, p. 2297-2301

141. d Rosen 5th, Chapter 164, p.2283-2287

142. e Rosen 5th, Chapter 67, p. 931-932

Neonatal conjunctivitis (ophthalmia neonatorum) occurs within the first month of life. The chemical irritation from antimicrobial prophylaxis against bacterial infection occurs within 1-2 days of birth. Other causes include Chlamydia trachomatis, H. influenzae, and Streptococcus pneumoniae. Due to mandated use of erythromycin ocular prophylaxis, Neisseria gonorrhoeae is not a major cause of neonatal conjunctivitis in the United States. This drug, however, has not been proven effective against chlamydia infections. Gonococcal conjunctivitis generally has its peak time of onset between 3 and 5 days after birth. By the end of first week of life and throughout the first month of life, chlamydia becomes the most frequent cause of conjunctivitis. Your next step should be a fluorescein staining of the reddened eye, followed by examination with a cobalt blue light. Corneal abrasion is a common cause of inconsolable crying in an otherwise normal infant. The infant with a history of nonbilious projectile vomiting must be considered to have pyloric stenosis. Physical examination usually demonstrates a hungry infant who has failed to gain weight over the past several weeks or has lost weight. Surgery is the treatment of choice although there are reports of success with IV and oral atropine, a Tetralogy of Fallot is one example of cyanotic heart disease associated with decreased pulmonary blood flow. Transposition of the great vessels exemplifies cyanotic heart disease with increased pulmonary blood flow. Other congenital cardiac lesions causing cyanosis are truncus arteriosus; tricuspid atresia; and total anomalous venous return. They are known collectively as the “terrible T’s.” A thorough and atraumatic cleansing of the ear canal is the most important part of therapy. For mild infections, dry mopping using a small tuft of cotton attached to a wire applicator is sufficient and may be curative. The tympanic membrane of a child with otitis externa may be as red and distorted as that of a child with otitis media, although mobility of the tympanic membrane is normal or only slightly decreased in otitis externa. Visualization of the tympanic membrane may be difficult because of edema of the external auditory canal.

53 143. Which statement is true regarding Group A beta-hemolytic streptococcal (GABHS) pharyngitis: a. Experienced clinicians can accurately exclude GABHS pharyngitis with the history and physical exam alone. b. Overuse of penicillin has resulted in emergence of resistant strains of GABHS requiring a change in recommended first line antibiotics. c. It is inappropriate to treat sore throat with antibiotics unless there is culture-proven GABHS. d. Throat culture for GABHS has very few (less than 1%) false-negatives. e. Treatment of streptococcal pharyngitis significantly shortens the course of the disease. 144. You are evaluating a 6 week-old infant for “irritability” as reported by her mother. You find an alert but cranky infant with a rectal temperature of 95oF. The left eardrum is a little red. The baby is fussy when held, but relatively quiet when left on the bed. Which statement is correct? a. This infant needs lumbar puncture. b. Since you found an otitis media, you have an explanation for the infant’s illness and an LP is not necessary. c. Given a reliable mother, antibiotic therapy and close outpatient follow-up is appropriate d. It is almost impossible for an infant this young to have an otitis media. e. If the fontanelles are flat, you can reliably rule out meningitis. 145. In a child with a chest x-ray that shows pneumonia, the most common physical finding would be: a. rales. b. rhonchi. c. tachypnea. d. asymmetric breath sounds. e. wheezes. 146. A 4-year-old girl presents with six days of fever. You find erythema of her hands and feet, conjunctival injection, cervical adenopathy, “strawberry” tongue, dry lips, red throat, and a morbilliform rash. You should treat this girl with: a. penicillin. b. aspirin. c. ceftriaxone. d. amoxicillin. e. dexamethasone. 147. A 3-year-old girl has a plastic bead stuck in her ear. She kicks and screams when you try to remove the bead and, even after sedation, you are unsuccessful in your attempts to remove the foreign body. Not wanting to cause injury, you give up and call the ENT surgeon, who tells you not to worry about it; send the patient home and have Mom call the next day for an appointment. You should: a. follow the consultant’s instructions. b. insist that the consultant come in and see the patient. c. call a different consultant. d. sedate the patient even more deeply to allow successful foreign body removal. e. turn the child upside-down and shake her vigorously. 148. Which electrical injury is correctly paired with its resultant complication? a. low-voltage alternating current (AC)  ventricular fibrillation b. lightning  ventricular fibrillation c. high-voltage AC  superficial burns d. lightning  compartment syndrome e. high-voltage AC  tetanic contraction

54

143. e th

Rosen 5 , Chapter 70, p. 971-972

144. a Rosen 5th, Chapter 168, p. 2344-2350

145. c Rosen 5th, Chapter 163, p. 2268-2270

146. b Rosen 5th, Chapter 161 p. 2238-2239

147. a Rosen 5th, Chapter 53 p.754-756

148. a Rosen 5th, Chapter 136, p. 2010-2014

Research has clearly demonstrated the beneficial effects of early antibiotic therapy on reduction of signs and symptoms of GABHS pharyngitis. Diagnostic accuracy on the basis of clinical findings alone is reported at about 50 to 75% for children thought to have GABHS and 75 to 85% for children thought not to have GABHS. There is general agreement that clinical diagnosis alone would result in an unacceptably high rate of misdiagnosis. GABHS is highly sensitive to penicillin, and there has been no evidence of development of resistance in vitro despite decades of use. The rate of false-negative results from single throat culture is about 10%. Because clinical judgment is insufficient and rapid diagnostic tests are not always accurate and diagnose only GABHS, this disease process is often treated empirically. The symptoms and signs suggestive of bacteremia and serious bacterial illness most frequently produce an overall ill appearance. Parents may note poor feeding, decreased responsiveness, or irritability in response to attempts to console. Physical examination findings suggestive of an ill appearance include poor eye contact and muscle tone, including weak suck, poor head control, and indifferent response to stimuli. The most important historical and physical findings are related to overall appearance. Bulging fontanelle, a hallmark of an increased intracranial pressure, is present in approximately 15% of neonatal acute bacterial meningitis. The best physical examination finding for ruling out pneumonia in an infant or child is the absence of tachypnea. However, tachypnea is a nonspecific symptom and may occur secondary to fever, anxiety, metabolic disease, cardiac disease, or other respiratory problems. Fever can increase an infant’s respiratory rate by 10 breaths per minute for each degree centigrade of elevation. Auscultation of the lungs may reveal localized rales, wheezing, and decreased air entry in the affected area. However, auscultatory findings may not be reliable in children. In younger children, decreased breath sounds, rather than rales, are often heard, since the involved areas tend to be ventilated poorly. Kawasaki syndrome is a vasculitis that causes a prolonged fever (more than 5 days), usually exceeding 39.5-40°C (104° F). Clinical findings include conjunctival injection without exudate; red, peeling lips; strawberry tongue; and pharyngitis. Early there may be painful swelling of the hands and feet; peeling of the skin of the fingers occurs late in the course. Cervical adenopathy in excess of 5 cm is common. Treatment is directed toward the amelioration of symptoms and the prevention of coronary aneurysms. Gamma-globulin 2 gm/kg intravenously should be administered over 12 hours, followed by high-dose aspirin therapy (100 mg/kg/24 hr PO given in divided doses every 6 hours for 14 days). If routine methods are unsuccessful or if the patient, is uncooperative or in too much distress, the emergency physician should cease removal efforts and refer the patient to an otolaryngologist. Primary operative intervention is frequently indicated in very young children and in those in whom the EP and the ENT specialist believe nonsurgical attempts unlikely to be successful. Inappropriately prolonged efforts at foreign object removal can result in wasted time, unnecessary patient discomfort, and high potential for complications. Low-voltage alternating current generally produces ventricular fibrillation. Highvoltage AC that causes greater current intensity is more likely to produce asystole. Direct current is also more likely to cause asystole. The most common arrhythmia encountered in victims who sustain cardiac arrest from electrical injury is ventricular fibrillation. The immediate cause of death due to a lightning strike is usually direct current depolarization of the myocardium and sustained cardiac asystole.

55 149. A 4-year-old non-immunized child presents with sudden sore throat, stridor, drooling, and fever. He looks sick and is sitting forward on the litter, but is alert. Pulse oximetry is 99% saturation on room air. Your next step is to: a. attempt to visualize his epiglottis. b. send him to x-ray for lateral neck radiographs. c. immediately begin bag-valve-mask ventilation. d. immediately intubate the child nasotracheally. e. none of the above. 150. A 3-year-old boy is brought to the emergency department after choking on a watch battery. X-rays show that the battery is in his stomach. Appropriate management involves: a. parental reassurance and home observation. b. ipecac to induce expulsion of the battery. c. cathartics to speed transit through the gastrointestinal tract. d. admission to the hospital for observation. e. laparotomy if the battery is still in the small bowel at 48 hours. 151. Which of the following fractures is most suggestive of child abuse? a. 19-month-old with a supracondylar fracture after falling off a chair. b. 22-month-old with a spiral femur fracture from a fall while running. c. 10-month-old with linear skull fracture from a fall down stairs in an infant walker. d. 4-month-old with a transverse fracture of the humerus from a fall out of bed. e. 3-year-old with a tuft fracture of the distal phalanx from the finger being closed in a door. 152. A 10-year-old boy complains of mild intermittent abdominal pain and a rash. His parents have not noted any fever. Physical examination shows a well-appearing boy with a raised purpuric rash, most prominent on the legs. His abdominal exam is normal. Temperature = 99.8oF. The other vital signs are normal. What is the most appropriate next step in the management of this patient? a. Obtain blood for culture and start antibiotics. b. Obtain abdominal radiographs. c. Obtain a urinalysis, BUN, and creatinine. d. Perform a skin biopsy. e. Order liver function tests. 153. Concerning cold exposure injuries: a. chilblains (pernio) is a painless macular rash. b. in frostbite, the “zone of stasis” is most severely injured. c. early surgical intervention is recommended for severe frostbite. d. a body part previously affected by cold injury is immune from reinjury. e. trench foot may result in irreversible damage. 154. The most common finding in a patient with a brown recluse spider bite is: a. vomiting. b. severe muscle cramps. c. anaphylaxis. d. local tissue necrosis. e. respiratory failure.

56

149. e th

Rosen 5 , Chapter 162

150. a Rosen 5th, Chapter 147, p. 2118

151. d th

Rosen 5 , Chapter 169, p. 2379-2381

152. c Rosen 5th, Chapter 167

153. e Rosen 5th, Chapter 133, p. 1974

154. d Rosen 5th, Chapter 55 p.796

The ideal approach is to take any patient with suspected epiglottitis to the operating room, administer anesthesia, and examine the airway with a laryngoscope while the patient is anesthetized. If the diagnosis of epiglottitis is made, the patient can be intubated. If it is ruled out, the patient can be returned to the ward or the emergency department to continue the workup, secure in the knowledge that epiglottitis is not present. Button batteries that have passed the esophagus need not be retrieved in the asymptomatic patient unless the cell is not passing through the pylorus after 48 hours of observation, in which case endoscopic retrieval is the preferred option. Ipecac has no place in the management of button battery ingestion. Fractures suggestive of child abuse include spiral fractures caused by torsion (twisting) of a long bone, and metaphyseal chip fractures, especially when present in infants less than 6 months of age. While this is the most suspicious of the fractures described, it must be emphasized that any of these fractures could result from abuse. Abuse must always be considered as a possibility with injuries involving young children. Henoch-Schönlein Purpura is an immunoglobulin A mediated systemic vasculitis involving the small blood vessels supplying the skin, gastrointestinal tract, and joints. The hallmark is a palpable, purpuric, or petechial rash most prominent on the lower extremities and extending to the buttocks, which is the presenting complaint in ~50% of patients. GI complaints are present in ~65% of patients, especially periumbilical, dull pain resulting from bleeding into the intestinal wall. 25% - 50% of children develop a self-limiting glomerulonephritis manifested by hematuria. There are no specific tests to confirm HSP, and the diagnosis can be difficult if the classic rash is absent. Screening tests such as urinalysis, BUN and creatinine, CBC, and coagulation studies may be needed to rule out other pathologic diseases. A helpful mnemonic is ARENA: A = Abdominal pain R = Rash E = Edema N = Nephritis A = Arthralgias / arthritis Trench foot develops slowly over hours to days and is initially reversible but if allowed to progress will become irreversible. Chilblains, or pernio, is characterized by mild but uncomfortable inflammatory lesions of the skin of bared extremities caused by chronic intermittent exposure to damp, nonfreezing ambient temperatures. The zone of stasis is the middle ground and is characterized by severe, but possibly reversible, cell damage. It is here that treatment is directed. Early surgical intervention is not indicated in the management of frostbite. Premature surgery has been an important contributor to unnecessary tissue loss and poor results in the past. Refreezing previously frozen tissue worsens the prognosis. Most victims of bites by the brown recluse spider do not even know they were bitten. Pain usually develops within 3 to 4 hours, and a white area of vasoconstriction begins to surround the bite. A bleb then forms in the center of this area, and an erythematous ring arises on the periphery. The lesion at this stage resembles a bull’s-eye. The bleb darkens, necroses over the next several hours to days, and continues to spread slowly and gravitationally, involving skin and subcutaneous fat.

57 155. A 25-year-old man was playing catch with his pet rattlesnake. He now complains of pain and swelling in his hand and forearm, with perioral numbness, and vomiting. His blood pressure is 90/60 mmHg. He will require: a. fluid resuscitation with fresh frozen plasma. b. administration of 2 to 4 vials of antivenin. c. measurement of coagulation factors and platelets. d. immediate forearm fasciotomy. e. non-narcotic pain medication, to avoid masking of respiratory symptoms. 156. Acetazolamide acts by inhibiting the enzyme carbonic anhydrase. This in turn: a. reduces reabsorption of bicarbonate in the kidney, leading to bicarbonate diuresis and metabolic acidosis, which produces compensatory hyperventilation. b. increases the blood carbon dioxide level, allowing a respiratory acidosis and compensatory diuresis. c. slows the sodium-potassium ATPase pump, causing a natriuretic compensation for the leftwardshifting oxygen dissociation curve. d. causes the kidney to reabsorb more bicarbonate ions, leading to a metabolic alkalosis, which shifts the oxygen dissociation curve to the right to compensate for lower atmospheric pressures. e. prevents acute mountain sickness by an unknown mechanism. 157. A 55-year-old male diver complains of back pain and urinary retention which started about one hour after ascent from a dive. He most likely has: a. bladder barotrauma. b. lumbar strain. c. envenomation by coelenterate species. d. nitrogen narcosis. e. decompression sickness. 158. Concerning High Altitude Pulmonary Edema (HAPE): a. it is the most lethal of the altitude illnesses. b. women are more susceptible than men. c. salt deprivation is a risk factor; use of a sleeping medication is somewhat protective. d. early symptoms include a moist cough with blood-tinged sputum; rales are universally present. e. the condition typically improves at night. 159. Choose the correct statement concerning electrical burns. a. When tissue carbonization occurs, resistance to current flow decreases even further. b. Contact with low voltage long distance communications lines and telephone can cause death in certain circumstances. c. A very narrow range exists between the threshold of perception of current and the level above which a person becomes unable to release the current source because of muscular tetany. d. The foot is the most common body part involved in alternating current injuries. e. Sweating increases resistance against electricity. 160. Which of these patients requires admission to a burn-care facility? a. A 35-year-old man with extensive partial-thickness burns on the back, shoulders, and buttocks. b. A 60-year-old diabetic with a full-thickness burn of the entire forearm. c. A 25-year-old woman with full-thickness burns of both hands and lower arms. d. A 40-year-old house-fire victim with multiple, small partial-thickness burns and wheezing. e. All of the above

58

155. c th

Rosen 5 , Chapter 55,

156. a Rosen 5th, Chapter 138, p. 2041

157. e Rosen 5th, Chapter 137, p. 2024

158. a Rosen 5th, Chapter 138, p. 2042

159. c Rosen 5th, Chapter 136, p.

160. e Rosen 5th, Chapter 56, p. 812, Box 56-2

Patients admitted to the hospital after poisonous snakebite should have serial determinations of platelets, prothrombin time, and urinalysis to check for myoglobin and hemoglobin. Daily comprehensive laboratory tests should be performed. Fasciotomy is not usually indicated unless compartment pressures are elevated. Antivenin (Crotalidae) polyvalent is the mainstay of therapy for poisonous snakebite; usually 10 vials or more are required. Debridement should probably not be performed earlier than 3 days after the bite, until the coagulopathy has resolved. Acetazolamide is a carbonic anhydrase inhibitor that induces a renal bicarbonate diuresis causing a metabolic acidosis, thereby increasing ventilation and arterial oxygenation. The diuretic effects may be of benefit for the fluid retention common in AMS. The drug also lowers CSF volume and pressure, which may play an additional role in its therapeutic and prophylactic use. The central nervous system is particularly susceptible to decompression illness because of its high lipid content. The spinal cord, especially the upper lumbar area, is more often involved than cerebral tissue. Symptoms of spinal DCS include limb weakness or paralysis, paresthesias, numbness, and low back and abdominal pain. Bladder symptoms, such as urinary retention, can occur, as well as fecal incontinence and priapism. High Altitude Pulmonary Edema (HAPE) is the most common fatal manifestation of severe high-altitude illness. It can occur, and even be fatal, at altitudes as low as 8000 feet. Many patients have a single episode of HAPE and subsequently are able to return to high altitude without a recurrence. Others who have had uneventful high-altitude exposures may have HAPE develop on a future ascent. A very narrow range exists between the threshold of perception of current (0.2 to 0.4 mA) and the let-go current (6 to 9 mA), the level above which a person becomes unable to release the current source because of muscular tetany. When tissue carbonization occurs, resistance to current flow increases. No deaths are recorded from contact with low voltages associated with long distance communications lines (24V) or telephone lines (65V). The hand is the most common site of contact via a tool that is in contact with an AC electric source. Sweating can reduce the skin’s resistance from 10 – 40,000 ohms to 2500-300 ohms. 1. Partial- or full-thickness burns involving >10% of body surface area (BSA) in patients under 10 or over 50 years of age. 2. Partial- or full-thickness burns of >20% of BSA in other age groups. 3. Partial- or full-thickness burns with the threat of functional or cosmetic impairment that involve face, hands, feet, genitalia, perineum, or major joints. 4. Full-thickness burns of >5% of BSA in any age group. 5. Electrical burns, including lightning injury. 6. Chemical burns with the threat of functional or cosmetic importance. 7. Inhalation injury with burns. 8. Circumferential burns of the extremities or chest. 9. Burn injury in patients with preexisting medical disorders that could complicate management, prolong recovery, or affect mortality. 10. Any burn patient with concomitant trauma, such as fracture.

59 161. Concerning victims of near-drowning: a. “dry drowning” without aspiration results from laryngospasm and glottal closure. b. electrolyte abnormalities contribute significantly to mortality. c. hemolysis resulting in anemia is found in more than half of fresh-water victims. d. disseminated intravascular coagulation is a common finding. e. postural drainage or the abdominal thrust (Heimlich maneuver) is recommended by most experts as a way to remove water from the lungs and improve oxygenation. 162. After radiation exposure, the best predictor of hematopoietic involvement is the: a. Absolute neutrophil count at 24 hours. b. Absolute neutrophil count at 48 hours. c. Absolute lymphocyte count at 24 hours. d. Absolute lymphocyte count at 48 hours. e. Absolute platelet count at 24 hours. 163. A 35-year-old man presents complaining of headache, weakness, nausea, and vomiting after working with paint remover in an enclosed space. You know that: a. a special antidote kit is required. b. treatment must continue longer in patients with this exposure than from other sources. c. the patient's oxygen-hemoglobin dissociation curve is shifted to the right. d. severe metabolic acidosis may be present. e. methylene blue may be required. 164. A 25-year-old photographer is found slumped over his workbench in the back of his store. Minutes earlier he had complained of headache, dizziness, and weakness. Presently the patient is comatose and apneic with a thready pulse. Cyanosis is absent. A venous blood sample drawn by paramedics in the field is noted to be very red. Laboratory results reveal a high anion gap acidosis. Blood gas determinations on arterial and venous blood samples reveal nearly identical pO2 values. Carboxyhemoglobin is 5%. Although the patient is intubated and being hyperventilated with 100% oxygen, the acidosis persists. What is the proper management? a. administer methylene blue IV b. administer physostigmine and hydroxocobalamin (vitamin B12a) c. administer 2-PAM d. administer sodium nitrite IV, then infuse sodium thiosulfate e. transfer the patient to a facility with a hyperbaric chamber 165. In a mass casualty situation involving a large number of victims of a “dirty bomb,” the highest priority patients are those: a. who are critically injured and contaminated. b. with external local body radiation only. c. with external total body radiation only. d. with internal contamination. e. with external contamination. 166. A 24-year-old man was bitten on the arm by his girlfriend’s cat six hours ago. He now has warmth and erythema at the site. The infecting organism is most sensitive to: a. gentamicin. b. metronidazole. c. clindamycin. d. penicillin. e. fluconazole.

60

161. a th

Rosen 5 , Chapter 139, p. 2051

162. d Rosen 5th, Chapter 140, p. 2058-2059

163. b Rosen 5th, Chapter 152, p. 2159-2162

164. d Rosen 5th, Chapter 153, p. 2166-2168

165. a Rosen 5th, Chapter 140, p. 2060-2062

166. d th

Rosen 5 , Chapter 54, p. 775-776

In a small number of submersion victims, severe laryngospasm causes hypoxia, convulsions, and death in the absence of active aspiration (“dry” drowning). Most drowning victims do not aspirate enough fluid to cause life-threatening changes in blood volume or serum electrolyte concentrations. Although DIC has been reported, it is uncommon. Procedures to drain fluid from the lungs are ineffective and potentially dangerous because of the increased risk of vomiting and aspiration. If particulate material is obstructing the airway, the Heimlich maneuver may be of benefit but it is not indicated for the removal of fluid. The absolute lymphocyte count 48 hours after exposure is a good predictor of hematopoietic involvement. If the absolute lymphocyte count is greater than 1200, it is unlikely that the patient has received a fatal dose of radiation. If the absolute lymphocyte count falls between 300 and 1200 at 48 hours, the possibility of exposure to a lethal dose of radiation should be suspected. A level in this range is an indication for hospitalization. Levels less than 300 are critical, and heroic procedures such as bone marrow transplant and the use of hematopoietic growth factors may be considered in individual cases. Carbon monoxide (CO) toxicity can develop when inhaled methylene chloride vapor in paint strippers or from leaking "bubble" electric Christmas tree lights is slowly metabolized to produce CO. The elimination half-life of CO from methylene chloride is about twice that of inhaled CO because it is stored in tissues and gradually released. The binding of CO to hemoglobin transforms the oxyhemoglobin dissociation curve from a sigmoid shape to an asymptotic shape, increasing the ability of HbCO to hold on to oxygen at the remaining heme moiety sites. In CO toxicity, both the reduced oxygen carriage and the transformation of oxyhemoglobin dissociation curve impair tissue oxygen delivery. In effect, high HbCO imposes the equivalent of a sudden “chemical” anemia in the patient. Cyanide in its salt form (e.g., sodium or potassium) is important in the metallurgic (e.g., jewelry) and photographic industries. Cyanide salts do not have an odor under dry conditions. Since cyanide prevents tissue extraction of oxygen from the blood, the oxygen content of venous blood approaches that of arterial blood. Clinically this may appear as the “arterialization,” or brightening, of venous blood to resemble arterial blood. Although the exact mechanism of the cyanide antidote kit is controversial, the accepted goal of therapy is to reactivate the cytochrome oxidase system by providing an alternative, high-affinity source of ferric ions (Fe3+) for cyanide to bind. Sodium nitrite and sodium thiosulfate are the active ingredients in the kit. If treatment of great numbers of radiation exposed and contaminated patients is necessary, resuscitation and stabilization always takes precedence over decontamination. However, health care workers must not expose themselves. Protection of health care workers from injury takes priority over the patient's ABCs'. Pasteurella multocida is the major pathogen found in infected cat bite wounds, isolated in up to 80%. Infection is characterized by a rapidly developing, intense inflammatory response, often within a few hours and rarely more than 24 hours after the bite. Pain and swelling are prominent. The antibiotic best used to treat the major organism in this presentation is penicillin.

61 167. Your friend has just stepped on a jellyfish while swimming. He is tearful because of the pain. You can offer temporary relief by: a. drying the area. b. rinsing the site with ethanol. c. rinsing the site with fresh water. d. rinsing the site with milk. e. rinsing the site with salt water. 168. The vast majority of mushroom-related fatalities in North America are due to: a. coprine-induced disulfiram-like reaction when consumed with ethanol. b. cyclopeptide-induced liver and renal failure. c. monomethylhydrazine-induced seizures and hepatorenal failure. d. muscarine-induced "SLUDGE" syndrome. e. psilocybin-induced hallucinations and seizures. 169. A 16-year-old girl ingested an unknown plant an hour ago. Now she is tachycardic with mydriasis, decreased bowel sounds, altered vision, and abnormal mental status. After ensuring the basic ABCs, your next action should be: a. intravenous atropine. b. GI decontamination. c. isotonic intravenous fluid administration. d. subcutaneous physostigmine. e. intramuscular glucagon. 170. A hiker on the Appalachian Trail is bitten on the hand by a snake, which was positively identified as having red bands next to yellow bands. She describes mild pain at the site of the bite, but is otherwise asymptomatic. Your recommended treatment is: a. admission to the hospital for observation only. b. discharge after extended observation. c. discharge after routine wound care. d. administration of antivenin if the patient becomes symptomatic. e. immediate administration of antivenin. 171. Which is the typical sequence of cardiac arrhythmic deterioration in a patient whose core temperature is below 30oC (86oF). a. atrial fibrillationventricular fibrillationpulseless electrical activityasystole b. sinus bradycardiaventricular tachycardiaventricular fibrillationasystole c. sinus bradycardiaatrial fibrillationventricular fibrillationasystole d. atrial fibrillationatrial flutterventricular fibrillationasystole e. sinus bradycardiaatrial flutteratrial fibrillationfine ventricular fibrillation 172. A 25-year-old man notices burning pain in his arm while retrieving logs from a woodpile. Within six hours, he has developed a necrotic area surrounded by an erythematous ring. He then notices fever, myalgias, nausea and generalized weakness. Of those listed, the most likely cause is: a. acute contact dermatitis. b. brown recluse spider bite. c. splinter from maple bark. d. deer tick bite. e. black widow spider bite.

62

167. e th

Rosen 5 , Chapter 55, p. 797-798

168. b th

Rosen 5 , Chapter 144, p. 2203-2205

169. b Rosen 5th, Chapter 144, p. 2081-2081 Chapter 158, p. 2199-2201

170. e Rosen 5th, Chapter 55, p. 789-792

171. c Rosen 5th, Chapter 134, p. 1989-1990

172. b Rosen 5th, Chapter 55, p. 795-796

Jellyfish envenomation sites should be washed off with salt water from the ocean, as fresh water is reported to worsen the pain.

Cyclopeptides such as amatoxin, found in certain amanita species, produce a threephase pattern of toxicity with early GI effects followed by a quiescent phase during which the patient may be released from medical care. The devastating third phase results in severe hepatic and sometimes renal toxicity. Ninety-five percent of North American deaths from mushroom poison are due to these toxins. While monomethylhydrazine poisoning from the Gyromitra species may have 10 – 40% mortality, this poisoning is much less common and thus yields fewer fatalities. Poisoning due to mushrooms containing muscarine, psilocybin, or coprine generally responds well to supportive measures and antidotal therapy. Anticholinergic toxicity commonly occurs with jimsonweed ingestion. This plant contains belladonna alkaloids such as atropine, scopolamine, and ecgonine. After assuring the ABC's, treatment includes GI decontamination with emesis or lavage, activated charcoal, and supportive care (IV fluids, external cooling, and restraints for patient protection). GI decontamination may be useful for up to 48 hours after ingestion if the patient remains symptomatic. Isotonic fluid administration is the first line treatment for hypotension. Many authorities recommend treatment of "full-blown" anticholinergic syndrome with physostigmine, but reports of asystole, ventricular arrhythmia, hypotension, bronchospasm, and seizures have occurred. “Red on yellow, kill a fellow; red on black, venom lack.” This applies to women victims, too. Anyone bitten by the Eastern coral snake (Micrurus fulvius) should be given the antivenin even before any symptoms develop. The toxicity of this venom has a rapid onset, and once the symptoms develop, it may be too late to reverse the effects with antivenin. The recommended dose is three to five vials in 300 to 500 ml of normal saline. Patients are at risk for dysrhythmias at body temperatures below 30°C (86°F); the risk increases as body temperature decreases. Although various dysrhythmias may occur at any time, the typical sequence is a progression from sinus bradycardia to atrial fibrillation with a slow ventricular response, to ventricular fibrillation, and ultimately, to asystole. As the name implies, the brown recluse spider is a reclusive organism. Humans encounter this spider in attics, storage sheds, crawl spaces, and woodpiles. Encounters with humans are uncommon. Brown recluse spiders are not aggressive and bite humans in self-defense. The venom results in epidermal and subcutaneous necrosis. This reaction is a local process around the bite site. About 6 to 8 h after the bite, pain associated with a red-to-violaceous discoloration develops.

63 173. When resuscitating a hypothermic patient whose core temperature is 14 mg/dl) require immediate treatment regardless of symptoms. The four basic goals of therapy are (1) restore intravascular volume, (2) enhance renal calcium elimination, (3) reduce osteoclastic activity, and (4) treat primary disorder. Isotonic saline is the first step. Once volume has been restored, the calcium will usually have decreased by 1.6 to 2.4 mg/dl, but hydration alone rarely leads to complete normalization. Loop diuretics such as furosemide inhibit the resorption of calcium in the thick ascending loop of Henle, increasing the calciuric effect of hydration. Volume expansion must precede the administration of furosemide, however, because the drug’s effect depends on the delivery of calcium to the distal nephron. Rapid fluid administration is the single most important initial step in the treatment of DKA. Fluid restores intravascular volume and normal tonicity, perfuse vital organs, improves glomerular filtration rate (GFR), and lowers serum glucose and ketones. The average adult patient has a water deficit of 100 mL/kg (5 to 10 L) and a sodium deficit of 7 to 10 mEq/kg. It is generally accepted that the “ideal way” to administer insulin is by continuous infusion of small doses of regular insulin through an infusion pump. This approach appears to be more physiologic, helps produce a more linear fall in serum glucose and ketone bodies, and is associated with less-severe metabolic complications (hypoglycemia, hypokalemia, and hypophosphatemia).

82 222. Euvolemic hyponatremia is found in patients with: a. diabetes insipidus. b. vomiting. c. laxative abuse. d. SIADH. e. decreased water intake. 223. A 27-year-old alcoholic with insulin dependent diabetes is lethargic and vomiting. He smells heavily of alcohol. Labs show Na+ 143, K+ 4.2, Cl- 107, HC03 18, glucose 305. His serum ethanol level is 0.128 (128 mg%). His urine dips positive for ketones. What is the likely cause of his ketoacidosis? a. alcoholic ketoacidosis, anion gap 36 b. diabetic ketoacidosis, anion gap 18 c. alcoholic ketoacidosis, anion gap 18 d. diabetic ketoacidosis, anion gap 36 e. diabetic ketoacidosis, anion gap 22 224. A common finding in myxedema is: a. hypocalcemia. b. hyponatremia. c. hyperglycemia. d. hypocholesterolemia. e. hypokalemia. 225. A 48-year-old woman with a history of Graves’ disease complains of nervousness and palpitations. Temperature 39.5oC; pulse 150 / minute; respirations 36 / minute; blood pressure 150/70 mmHg. She is thin and very anxious. The first medication you should give is: a. propylthiouracil. b. dexamethasone. c. potassium iodide (SSKI). d. methimazole. e. propranolol. 226. A 48-year-old woman with a history of rheumatoid arthritis complains of profound weakness and diffuse abdominal pain. She just returned from a two week cruise and admits that she forgot to take her prednisone. You suspect adrenal insufficiency and begin her treatment with: a. potassium supplementation. b. rapid volume replacement. c. intramuscular mineralocorticoid. d. subcutaneous insulin. e. colloid volume expanders. 227. A 73-year-old man with a history of noninsulin-dependent diabetes mellitus is being evaluated for altered mental status. Physical exam is remarkable for signs of severe dehydration. His serum glucose level is 1283 mg/dl. You also expect to find: a. BUN : creatinine ratio less than 20:1. b. markedly elevated serum ketones. c. elevated troponin. d. hyponatremia (uncorrected).

83 e. profound acidosis.

84

222. d th

Rosen 5 , Chapter 119, p. 1724-1725

223. b Rosen 5th, Chapter 179, p. 2524

224. b Rosen 5th, Chapter 122, p. 1776, Box 122-9

225. e Rosen 5th, Chapter 122 p.1774

226. b Rosen 5th, Chapter 122, p. 1781-1782

227. d Rosen 5th, Chapter 120, p. 1755

The many causes of euvolemic hyponatremia include the syndrome of inappropriate secretion of ADH (SIADH), defined as the secretion of ADH in the absence of an appropriate physiologic stimulus. Hypovolemic hyponatremia results from the loss of water and sodium with a greater relative loss of sodium. Typical causes include vomiting, diarrhea, gastrointestinal (GI) suction or drainage tubes, fistulas, and “third spacing” of fluids (e.g., burns, intra-abdominal sepsis, bowel obstruction, pancreatitis). The difference between the serum sodium (the contribution of potassium, largely an intracellular ion, is usually ignored) and the sum of serum chloride and bicarbonate equals the concentration of the unmeasured anions, or the "anion gap." Almost all patients with DKA present with blood glucose greater than 300 mg/dL, but patients who present just after receiving insulin or who have impaired gluconeogenesis (e.g., in alcohol abuse or liver failure) may have lower initial serum glucose levels. In alcoholic ketoacidosis, alcohol levels are usually low or undetectable at the time of presentation, but some patients may present with an elevated blood alcohol level, making diagnosis more challenging. Serum glucose levels are usually less than 200 mg/dl. In the myxedematous patient, hyponatremia occurs often and is usually mild. The mechanism is thought to be syndrome of inappropriate secretion of antidiuretic hormone (SIADH), and thyroid replacement therapy reverses the abnormality. Hypoglycemia is unusual and typically mild; its correction usually does not materially affect the clinical symptoms. Hypercalcemia is rare, mild when present, and of uncertain cause. Cholesterol levels are typically elevated, are rarely less than 250 mg/dl, and in 86% of cases are greater than 290 mg/dl. A mild normocytic, normochromic anemia without reticulocytosis may be present. Treatment of thyroid storm has five goals: (1) inhibit hormone synthesis, (2) block hormone release, (3) prevent peripheral conversion of T4 to T3, (4) block the peripheral effects of thyroid hormone, and (5) provide general support. Blockade of the peripheral adrenergic hyperactivity of thyroid crisis may be the most important factor in reducing morbidity and mortality. Treatment of adrenal crisis includes replacement of fluids and sodium, administration of glucocorticoid, correction of hypotension and hypoglycemia, reduction of hyperkalemia, and identification and treatment of a precipitating cause of the crisis. A rapid infusion of 5% dextrose and isotonic saline should be started immediately. This acts to correct dehydration, hypotension, hyponatremia, and hypoglycemia. Patients in a hyperglycemic hyperosmolar nonketotic coma have a blood glucose level greater than 600 mg/dl and serum osmolarity greater than 350 mOsm/L. The BUN concentration is invariably elevated. Although patients with HHNC do not have a ketoacidosis caused by diabetes, they may have a metabolic acidosis secondary to some combination of lactic acidosis, starvation ketosis, and retention of inorganic acids attributable to renal hypoperfusion. They typically manifest more profound electrolyte imbalance than with DKA. Initial serum sodium readings will be inaccurately low because of hyperglycemia.

85 228. A 25-year-old with hemophilia A has a painful swollen ankle but no history of trauma. Appropriate treatment includes ice, immobilization, and a Factor VIII infusion of: a. 12.5 units/kg. b. 25 units/kg. c. 40 units/kg. d. 50 units/kg. e. 100 units/kg. 229. A 4-year-old boy has fever and headache. Temperature 104°F, pulse 168 / minute, respiratory rate 42 / minute, blood pressure 58/35 mmHg. He is lethargic and irritable, and has a purpuric rash. You start antibiotics and steroids and perform a lumbar puncture, which shows Gram-negative diplococci. One hour later, the nurse notes that there is oozing from his IV site. He then has a large bloody stool and passes dark red-brown urine which tests strongly positive for blood. You expect lab studies to show: a. elevated fibrinogen. b. elevated fibrin degradation products (FDP) and d-dimers. c. diminished INR. d. elevated platelet count. e. shortened thrombin clot time. 230. A 72-year-old woman with a past history of congestive heart failure complains of epigastric pain, black bowel movements, and vomiting some “coffee-ground” material. Her hemoglobin is 6.5 mg/dL, so you order two units of packed red blood cells for transfusion. The first unit is transfused uneventfully over a 45 minute period. Thirty minutes after the second unit of packed red blood cells is started, she complains of a headache and shortness of breath. Her lung exam now shows bilateral crackles. You tell the nurse to: a. give a bolus of intravenous saline. b. use a leukocyte reduction filter. c. administer diphenhydramine. d. slow the transfusion and begin diuresis. e. stop the transfusion and report a transfusion reaction. 231. A healthy 12-year-old African-American female complains of weakness and fatigue 3 days after starting a course of trimethoprim-sulfamethoxasole and pyridium for a urinary tract infection. Her hemoglobin is 4.8 mg/dl, and her urine is tea-colored, but you see no red blood cells on microscopic exam. She probably has undiagnosed: a. hemolytic uremic syndrome. b. G6PD deficiency. c. idiopathic thrombocytopenic purpura. d. thrombotic thrombocytopenic purpura. e. sickle cell disease. 232. In methanol poisoning the most likely cause of toxicity is: a. acetaldehyde. b. formic acid. c. folate. d. ethyl methanol. e. oxalic acid

86

228. b th

Rosen 5 , Chapter 116, p.

Treatment of nontraumatic hemarthrosis in a hemophiliac patient begins DDAVP. If not available, Factor VIII 12.5 U/kg as a single dose is given for early or mild bleeds; most hemophiliacs usually require 25 U/kg every 24 hours for 2 or 3 days. Results

229. b Tests for DIC

Pathophysiology

th

Rosen 5 , Chapter 116, p. 1698, Table 116-3

Peripheral smear

Low platelets, schistocytes, RBC fragments

RBCs fragment on fibrin strands (schiztocytes not always seen)

Platelet count

Low – usually 4 grams (in children >75 mg/kg) acetaminophen in a 24 hour period associated with malnourishment, chronic alcohol consumption, or chronic use of medications that induce the cytochrome P450 system. Laboratory evaluation should also be performed for adults who ingest >7.5 grams (in children >150mg/kg) in a 24 hour period. Once acetaminophen (APAP) is absorbed, hepatic metabolism normally accounts for up to 90% of its elimination. APAP is primarily metabolized in the liver through three routes: (1) conjugation with glucuronide (40% to 67%), (2) conjugation with sulfate (20% to 46%), or (3) oxidation via the cytochrome P450 (CYP450) mixed-function oxidase system with subsequent conjugation. The oxidation of APAP by CYP450 subfamilies results in the formation of highly reactive NAPQI. NAPQI rapidly combines with glutathione and other thiol-containing compounds forming nontoxic conjugates, which are eliminated in the urine. When NAPQI formation exceeds glutathione supply, free NAPQI binds to hepatocyte intracellular proteins, causing toxicity. At 6 weeks of gestation, the gestational sac can be visualized with transabdominal sonography (TAS). At 8 weeks, the fetal pole and fetal heart activity can be visualized using transabdominal sonography (TAS). Using transvaginal sonography (TVS) an intrauterine gestational sac can be seen at 5 weeks gestation and this correlates with an HCG of more than 1,800 IU/L. Fetal heart motion can be detected using transvaginal sonography (TVS) at 6 weeks of gestation and correlates with an HCG level of 6,770 IU/L. Ultrasound is the most useful test for evaluating gallbladder disease in the emergency department It is the procedure of choice in evaluating for gallstones. Visualization of the gallbladder without identification of stones has an extremely high negative predictive value for cholecystitis, whereas the presence of stones, a thickened gallbladder wall, and pericholecystic fluid has a positive predictive value in excess of 90%.

103 277. The FAST ultrasound exam: a. is for the evaluation of ectopic pregnancy. b. has the primary role of detecting free retroperitoneal blood after blunt trauma. c. is sensitive for detecting subcapsular splenic injury. d. may be false positive in a patient with ascites. e. .is sensitive for bowel perforations. 278. Usually, the minimal amount of intraperitoneal fluid that ultrasound (US) easily detects is: a. 10 ml. b. 50 ml. c. 500 ml. d. 1,000 ml. e. 1,500 ml. 279. An anatomic area not well visualized by CT scan is the: a. ureter. b. retroperitoneum. c. aorta. d. bladder. e. posterior intracranial fossa. 280. You are called to the radiology suite when an outpatient having an IVP done suddenly becomes short of breath, tachycardic, and complains of chest pain. His face is red, you hear wheezing as you walk into the room, and the nurse tells you his blood pressure is 76 palpable. The most important treatment is: a. diphenhydramine (Benadryl®) 50 mg IV. b. cimetidine (Tagamet®) 300 mg IV. c. methylprednisolone (Solu-Medrol®) 125 mg IV. d. epinephrine 10ml of 1:100,000 dilution IV over 10 minutes. e. albuterol by nebulizer. 281. The procedure of choice to evaluate possible aortic dissection in the unstable patient is: a. magnetic resonance imaging. (MRI). b. transthoracic echocardiography (TTE). c. transesophageal echocardiography(TEE). d. computed tomography (CT). e. aortography. 282. MRI is the procedure of choice in the acute evaluation of: a. spinal epidural hematoma. b. periorbital/globe injury. c. bladder injury. d. traumatic bowel perforation. e. testicular torsion. 283. A 8-month-old girl has scabies. The preferred treatment is: a. crotamiton (Eurax®). b. Diphenhydramine (Benadryl®). c. lindane (Kwell®). d. permethrin (Elimite®). e. topical hydrocortisone.

104

277. d th

Rosen 5 , Chapter 30, p. 252-253, Chapter 39, p. 423-424

278. c th

Rosen 5 , Chapter 39, p. 423

279. e th

Rosen 5 , Chapter 34, p. 296

280. d th

Rosen 5 , Chapter 113, p. 1632

281. c Rosen 5th, Chapter 80, p. 1173-1174

282. a Rosen 5th, Chapter 36, p. 360,363, fig 36-40 Chapter 100, p. 1502-1503

283. d th

Rosen 5 , Chapter 114, p. 1651

Focused assessment with sonography for trauma (FAST) exam’s primary role is detecting free intraperitoneal blood after trauma. . It cannot distinguish blood from ascites. False-negative FAST scans commonly involve bowel perforations or solid parenchymal damage if free intraperitoneal blood is absent as in subcapsular splenic injury.

Sensitivity in detecting as little as 100 ml and, more typically, 500 ml of intraperitoneal fluid ranges from 60% to 95% using ultrasound.

MRI is better than a head CT in detecting posttraumatic ischemic infarctions, subacute nonhemorrhagic lesions and contusions, and axonal shear injury, and lesions in the brainstem or posterior intracranial fossa. MRI is not as useful as CT in detecting bony injuries or hyperacute bleeding. Epinephrine, with its combined - and - adrenergic agonist actions, is the first drug of choice in the treatment of anaphylaxis. The route of administration chosen depends of the severity of the clinical presentation. Subcutaneous epinephrine is usually effective in those situations in which the clinical manifestations are mild and the patient is normotensive. In the patient with diffuse, generalized uriticaria, subcutaneous absorption of epinephrine may be slow and unpredictable, so the intramuscular route should be used. If the patient demonstrates severe upper airway obstruction, acute respiratory failure, or shock (systolic blood pressure less than 80 mm Hg, not associated with a ventricular tachydysrhythmia), intravenous epinephrine should be administered. The risk of supraventricular, accelerated idioventricular, and ventricular tachydysrhythmias; accelerated hypertension; and myocardial ischemia, including the stunned heart syndrome, is increased by using intravenous route of epinephrine. Because of these risks, dilution and slow administration is recommended. While CT, MRI, and aortography have all been used to diagnose aortic dissections, they all require the patient to leave the ED. In contrast transesophageal echocardiography (TEE) may be done at the bedside and thus when available is the primary diagnostic method for evaluating for possible aortic dissection in the unstable patient. Transthoracic echocardiography (TTE) has not been useful in detecting aortic dissection. The primary advantages of MRI over other imaging modalities lie in its ability to directly image nonosseous structures of the spinal complex. An MR scan allows noninvasive direct visualization of intramedullary and extramedullary spinal abnormalities that potentially cause neurologic deficit. Its major impact has therefore been in demonstrating lesions that are potentially correctable surgically, including acute disk herniation, ligamentous injury, bony compression, epidural and subdural hemorrhage or abscess, and vertebral artery occlusion. Treatment options for scabies include crotamiton (Eurax®) lotion and cream, or permethrin 5% cream (Elimite®). Lindane is no longer the preferred treatment. Patients in whom the former treatment fails may respond to the latter. Permethrin 5% cream applied overnight once weekly for 2 weeks over the entire body is the treatment of choice for infants and small children. It is more effective than crotamiton in eliminating the mite, in reducing secondary bacterial infection, and in reducing pruritus.

105 284. A 20-year-old woman complains of painful, swollen right and left knees , low grade fever, and a pustular rash on her hands. Your next step in her evaluation is: a. ASO titer. b. pelvic exam. c. knee ultrasound. d. rheumatoid factor. e. urinalysis. 285. A 57-year-old female complains of oozing lesions with mild swelling and slight erythema of her face and hands after working in her garden 2 days prior. The appropriate treatment of moderate to severe dermatitis on the face and hands from a known poison ivy exposure is: a. dexamethasone (Decadron®) injection 0.01 mg/kg. b. intravenous hydrocortisone (Solu-Cortef®). c. prednisone 10 mg daily for 10 days. d. prednisone tapered over 21 days, starting with relatively high dose. e. topical calamine lotion. 286. A 54-year-old man has had a burning pain in his chest for 5 days. He has no shortness of breath, and there is no radiation. Today he noticed this rash, which alarmed him and his wife. You can tell them that this rash is associated with: a. a high mortality rate. b. arthralgia. c. neuralgia. d. pruritus. e. resolution in 48-72 hours.

Rosen 5th Figure 114-29

287. Nikolsky's sign is characteristically seen in: a. toxic shock syndrome. b. pemphigus vulgaris. c. Rocky Mountain spotted fever. d. scarlet fever. e. erythema nodosum 288. A 17-year-old female complains of large raw, denuded areas of her skin. She also has low grade fever and generalized malaise. She does recall seeing her doctor recently for an upper respiratory infection. Toxic epidermal necrolysis (TEN): a. has no association with immunizations. b. has mucus membrane involvement . c. has a low mortality rate, generally less than 2%. d. is rarely caused by medications. e. causes bullae along the cleavage plane between the dermis and the subcutaneous tissue. 289. The treatment of contact dermatitis includes: a. cool wet compresses of Burow´s solution. b. topical corticosteroid ointment. c. systemic steroids. d. systemic antihistamines.

106 e. all of the above.

107

284. b th

Rosen 5 , Chapter 110, p. 1593

285. d Rosen 5th, Chapter 114, p. 1647-1648

286. c Rosen 5th, Chapter 114, p. 1656

287. b Rosen 5th, Chapter 114, p. 1643,1653 Chapter 131, p. 1946

288. b th

Rosen 5 , Chapter 114, p. 1643

289. e Rosen 5th, Chapter 114, p. 1647-1648

Gonococcal arthritis begins with fever, chills, and a migratory tenosynovitis and arthralgias that progress to arthritis, predominantly in the knee, ankle, or wrist. A characteristic rash in two thirds of patients accompanies the tenosynovitis and arthritis—a countable number of hemorrhagic necrotic pustules that typically first appear on the distal extremities, including the sides of the fingers. Rarely does the patient complain of cervicitis or urethritis. Cervical, urethral, rectal, and pharyngeal cultures are positive in 75% of cases. For best yield, all possible sites (synovium, blood, cervix, urethra, rectum, pharynx, and skin lesions) should be cultured. The initial diagnosis is a clinical one; it is the diagnosis to be excluded in any young patient with a fever, migratory polyarthritis, and polytendinitis. Oozing or vesiculated lesions caused by the Rhus species (poison ivy ,oak, and sumac) should be treated with cool wet compresses of Burow’s solution. Topical baths, available over the counter, may also be comforting. A course of systemic corticosteroids is often necessary. Prednisone in a dosage of 30 to 80 mg/day (depending on the severity of involvement) should be prescribed initially. This should be tapered over at least 10 to 14 days, and 21 days for poison ivy. Herpes zoster has a very low mortality rate and is rarely life threatening even when dissemination to the visceral organs occurs. The vesicles progress to scab then crust over 10 to 12 days then the scab falls off in 2 to 3 weeks. Pain is the symptom typically reported. Complications include CNS involvement, ocular infection, and neuralgia. Meningoencephalitis, myelitis, and peripheral neuropathy have been reported. Postherpetic neuralgia, pain that persists after the lesions have healed, occurs more commonly in the elderly and immunosuppressed patients. It may last a number of months and is often resistant to treatment with standard analgesics. Nikolsky’s sign is the easy separation of the outer portion of the epidermis from the basal layer (desquamation) when pressure is exerted. It is present in and characteristic of pemphigus vulgaris. It is also found in .toxic epidermal necrolysis (TEN) and staphylococcal scalded skin syndrome.

The main feature of nonstaphylococcal-induced toxic epidermal necrolysis (TEN), or Lyell’s disease, is the separation of large sheets of epidermis from underlying dermis. The full thickness of epidermis is involved. Drugs including the long-acting sulfa drugs, penicillin, aspirin, barbiturates, phenytoin, carbamazepine, allopurinol, and nonsteroidal anti-inflammatory drugs are an important cause of TEN. TEN has occurred after vaccination and immunization against polio, measles, smallpox, diphtheria, and tetnus. A mortality rate of 15% to 20% is expected with this condition. Mucus membrane involvement is the rule. Treatment of contact dermatitis includes avoidance of the irritant or allergen and treatment of secondary bacterial infection. Oozing or vesiculated lesions should be treated with cool wet compresses of Burow’s solution applied for 15 minutes three or four times a day. Topical baths, available over the counter, may also be comforting. A course of systemic corticosteroids is often necessary. Prednisone in a dosage of 30 to 80 mg/day (depending on the severity of involvement) should be prescribed initially. This should be tapered over at least 10-14 days, and 21 days for poison ivy. The long, slow taper is needed to prevent rebound of the disease. Systemic antihistamines, such as hydroxyzine and diphenhydramine, may help control pruritis. The patient should also be counseled to wash all clothes that might have contacted the plant.

108

290. A 27-year-old male presents with the lesions shown, but does not recall any pertinent historical information. A linear vesicular eruption on exposed area of the arms or legs is most likely secondary to: a.diabetic neuropathy. b.mycobacterium. c.parasite. d.poison ivy. e.spirochete.

Rosen 5th Fig 114-12

291. A Tzanck smear can establish a diagnosis of: a. gonorrhea. b. herpes. c. syphilis. d. staphylococcal cellulites. e. candida. 292. A 4-year-old child presents with a patch of scaly alopecia for two weeks duration. Recommended treatment would be: a. clotrimazole (Lotrimin®) cream. b. griseofulvin oral suspension. c. mupirocin (Bactroban®) ointment. d. systemic steroids. e. cephalexin. 293. A 65-year old woman complains of blisters on her face over the last two days which are quite painful. She has no other complaints. Physical examination reveals the rash shown here. Which of the following examinations is also essential for appropriate evaluation? a. sedimentation rate b. complete blood count c. sinus MRI d. lumbar puncture e. slit lamp examination 294. Appropriate treatment of neuroleptic malignant syndrome includes: a. antihistamines (both H1 and H2). b. anticholinergics. c. high-dose steroids. d. dantrolene. e. physostigmine. 295. Which statement about suicide is true? a. Attempted suicide rates are equal in both sexes. b. Completed suicide rates are equal in both sexes. c. Marital status does not seem to have an effect on successful completion rates. d. Older individuals are more likely to complete or succeed.

109 e. Wrist cutting is the most common attempted method of suicide.

110

290. d th

Rosen 5 , Chapter 114, p. 1647

291. b Rosen 5th, Chapter 93, p. 1392

292. b th

Rosen 5 , Chapter 114, p. 1635-1636

293. e Rosen 5th, Chapter 114, p. 1656

294. d Rosen 5th, Chapter 155, p. 2177-2178

295. d Rosen 5th, Chapter 109, p. 1576

The primary lesions of contact dermatitis are papules, vesicles, or bullae on an erythematous bed. Of the allergens, Rhus species are the most likely to cause bullous eruptions. Oozing, crusting, scaling, and fissuring may be found, along with lichenification in chronic lesions. The distribution of the eruption depends on the specific contactant and may be localized, asymmetric linear, or unilateral. HSV infections are often obvious, but if tehre is doubt or a need to confirm the diagnosis, a simple Tzanck slide preparation helps reveal the characteristic multinucleated giant cells approximately 50% of the time. Tinea capitis is a dermatophyte infection of the scalp. It is most commonly seen in children, particularly African-American children. Clinically one sees areas of alopecia with broken-off hairs and scale at the periphery. The alopecia is patchy and usually nonscarring. Occasionally, tinea capitis is associated with an intense inflammatory response. After a diagnosis is established, the current first-line therapy is griseofulvin. Topical treatment alone is not effective. Herpes zoster, or shingles, is an infection caused by the varicella-zoster virus. It occurs exclusively in individuals who have previously had chicken pox. Ocular complications occur in 20% to 70% of the cases involving the ophthalmic division of the trigeminal nerve. Eye involvement produces anterior uveitis, secondary glaucoma, and corneal scarring. There is a close correlation between eye involvement and vesicles located at the tip of the nose (Hutchinson’s sign). Treatment of neuroleptic malignant syndrome revolves around supportive care and discontinuation of the offending medication. Agitation, psychomotor hyperactivity, and muscular rigidity should be treated aggressively with intravenous benzodiazepines. Dantrolene, which inhibits the release of calcium from the sarcoplasmic reticulum, has been advocated as an adjunctive therapy for NMS, along with the dopamine antagonists bromocriptine and amantadine. There is no proven benefit to any of these agents. Suicides are highest among older individuals, particularly elderly white men. White men commit 73% of all suicides in the United States. Whites and Native Americans are much more likely to commit suicide than African Americans, Hispanics, or Asians. Marriage decreases the likelihood of suicide, but separated or divorced persons have a higher rate of suicide than those who never had a close relationship. Women attempt suicide three to four times more often than men, whereas men are three to four times more likely to kill themselves. Men have a higher incidence of alcoholism and tend to use more lethal methods, such as firearms. Most completed suicides involve firearms (70%), whereas most attempted suicides involve the ingestion of drugs or poisons (72%).

111 296. A 70-year-old male complaints of difficulty sleeping followed by periods of sleeping for over 20 hours daily, fatigue, difficulty concentrating and forgetfulness. He denies weight loss or new weaknesses. Upon further questioning, you find that his wife of 45 years passed away 4 months ago. The most likely cause of these symptoms is: a. cancer. b. dementia. c. depression. d. hypothyroidism. e. stroke. 297. Symptoms of major depression include: a. sleep disturbances. b. dysphoria. c. agitation. d. lack of concern about personal hygiene. e. all of the above. 298. A 52-year-old schizophrenic woman has been treated with fluphenazine (Prolixin®) for many years. Today she complains of uncontrollable movements of her face and tongue. She is probably suffering from: a. acute psychotic break. b. akathisia. c. dystonia. d. neuroleptic malignant syndrome. e. tardive dyskinesia. 299. The most appropriate management of a comatose patient who overdosed on his lithium is: a. activated charcoal. b. adenylate cyclase stimulators. c. chelation with EDTA. d. hemodialysis. e. aggressive fluid replacement. 300. A 44-year-old young woman complains of the sudden onset of blindness. She was visiting her debilitated father in the hospital. Her husband confides that she is distraught by her father’s diagnosis of metastatic colon cancer and the sight of him distresses her greatly. Her physical exam, including pupillary response and retinal exam is completely normal. Which statement is true regarding her likely diagnosis? a. The disorder usually presents with multiple complaints. b. It is more prevalent in well educated and higher socioeconomic social status. c. It usually involves orthopedic or neurologic complaints. d. The patient is typically older. e. Patients are typically very distressed by their symptoms.

112

296. c 297. e Rosen 5th, Chapter 105, p. 1550-1551

298. e Rosen 5th, Chapter 155, p. 2175

299. d Rosen 5th, Chapter 154, p. 2173

300. c Rosen 5th, Chapter 107, p. 1566

The essential features of major depression are a persistent dysphoric (sad) mood or pervasive loss of interest in usual activities, lasting for at least two weeks. “Vegetative symptoms” involve physiologic functioning and include loss of appetite and weight, sleep disturbance, fatigue, inability to concentrate, and psychomotor agitation or retardation. The depression may begin gradually or rapidly but usually will have been present for several weeks before the patient comes for treatment. Tardive dyskinesia is an involuntary movement disorder that is idiosyncratically associated with chronic therapy with the typical antipsychotic agents. Symptoms classically include involuntary, repetitive, choreoathetoid movements of the face, mouth, and tongue, and may be mild or severe and debilitating; uncommonly, the limbs and trunk are also affected. Elderly patients, especially those with diabetes mellitus and organic brain damage, seem to be at increased risk. The overall incidence is approximately 5% per year of exposure to the offending agent. Severely lithium-poisoned patients in coma or with seizures may require urgent airway interventions and anticonvulsant therapy with benzodiazepines, phenobarbital, and propofol as necessary. Many of these patients will be volume depleted. Although frank hypotension is unusual, fluid resuscitation is needed and should include normal saline. The goal of saline administration is to restore glomerular filtration, normalize urine output, and enhance lithium clearance. Forced diuresis does not assist in lithium clearance. Dialysis may be useful for patients with an acute overdose who have decreased level of consciousness, seizures, or levels greater than 4.0 mEq/L, although asymptomatic patients with higher levels have been managed without dialysis. For chronic toxicity, dialysis is helpful in symptomatic patients with levels greater than 2.5 mEq/L. Also known as hysterical neurosis, conversion type, the rare conversion disorder is characterized by the sudden onset and dramatic presentation of a single symptom, typically simulating some nonpainful neurologic disorder for which there is no pathophysiologic or anatomic explanation. The most common ages of presentation are adolescence or early childhood, although other age groups are also affected. The symptoms, generally conforming to the patient’s own idiosyncratic ideas about illness, are not under the patient’s voluntary control. The most common presentations to the ED include pseudoseizures, syncope or coma, and paralysis or other movement disorders. Most patients are women, except for military service and industrial accidents. Symptoms tend to be of sudden onset, waxing and waning in response to environmental stresses. Patients may describe their conversion symptoms with a lack of appropriate concern about their profound bodily dysfunction (la belle indifférence).

113 301. You are treating a 2-year-old boy who is in cardiac arrest. Your boss borrowed the Broselow tape for a paramedic lecture, but that’s okay because you know that: a. atropine is the first drug given for bradycardia in all age groups. b. the minimum dose of atropine is 0.1 mg. c. the initial defibrillation energy is 5 j/kg. d. the curved Macintosh blade will be more effective than the straight Miller when you are intubating. e. the tidal volume necessary to ventilate children is 5 to 8 ml/kg. 302. Which statement about appendicitis in the pediatric age group is true? a. Perforation and peritonitis are uncommon in younger children. b. The presence of diarrhea effectively excludes appendicitis. c. The condition most commonly mistaken for acute appendicitis is mesenteric adenitis. d. Most abdominal x-rays in patients with appendicitis reveal a fecalith e. The peak age of incidence in appendicitis is 6 to 10 years of age. 303. An Apt test is apt to: a. differentiate central causes of blindness from peripheral causes. b. confirm knee stability. c. differentiate seizures from pseudo-seizures. d. differentiate maternal from fetal blood. e. determine the presence of pathogenic bacteria in stool samples. 304. A 7-year-old boy has fever, abdominal pain, and vomiting over the past 18 hours. He has pain in his right lower abdomen when he coughs, and you find McBurney’s point to be exquisitely tender. Suspicious that he has a perforated appendix, you page the surgeon and instruct the nurse to give the patient: a. ceftriaxone (Rocephin®). b. ampicillin and gentamicin. c. ampicillin, gentamicin, and metronidazole (Flagyl®. d. a macrolide and a fluoroquinolone. e. levofloxacin (Levaquin®). 305. Hirschsprung's disease: a. accounts for 50% of intestinal obstruction in young infants. b. requires barium enema to make the diagnosis. c. is more frequently found in females. d. is usually diagnosed early in infancy. e. usually presents with a massive GI bleed. 306. The most common cause of intestinal obstruction between 3 months and 6 years of age is: a. cystic fibrosis. b. Hirschsprung’s disease c. intussusception d. Meckel’s diverticulum. e. natal adhesions.

114

301. b th

Rosen 5 , Chapter 8, pages listed in text

302. c Rosen 5th, Chapter 165, p. 2311-2312

303. d Rosen 5th, Chapter 165, p. 2307

304. c Rosen 5th, Chapter 88, p. 1298

305. d Rosen 5th, Chapter 165, p. 2305-2306

306. c Rosen 5th, Chapter 165, p. 2304

The recommended dose of atropine is 0.02 mg/kg IV, with a minimum dose of 0.1 mg to prevent paradoxical bradycardia (p. 89). Initially, 2 joules (watts per second)/kg should be used for defibrillation (p. 97). Epinephrine is indicated in all cardiac arrest settings, including asystole, pulseless electrical activity, and ventricular fibrillation (p.84). Because of the superior and anterior position of the tracheal opening, as well as the large tongue and relatively large floppy epiglottis, visualization of the glottic opening can be obstructed in children less than 4 years of age if a curved blade is used. The straight blade lifts these tissues so the glottic opening can be recognized (p. 91). The tidal volume necessary to ventilate children is the same as that for adults: 10 to 15 mL/kg (p. 93). The condition most commonly mistaken for acute appendicitis in children is mesenteric adenitis. The rate of appendiceal perforation before surgery varies from 17% to 40% and is inversely related to age with higher rates in younger age groups. The presence of diarrhea does not rule out appendicitis. An appendicolith is found in about 10% of plain films. The peak age of incidence in appendicitis is 9 to 12 years of age. In young neonates an Apt test can be performed to determine if a bloody stool contains maternal or fetal blood. Add 1% NaOH to the bloody stool. Fetal hemoglobin resists oxidation and remains pinkish-red, whereas maternal hemoglobin changes to a dark brown color. Once the decision has been made to operate, prophylactic antibiotics should be given to cover gram-negative and anaerobic organisms. Intravenous second-generation cephalosporins, such as cefotetan of cefoxitin, provide good coverage. If the appendix has already perforated, more complete coverage with ampicillin, gentamicin, clindamycin, or advanced penicillin combinations such as piperacillin and tazobactam may be preferred. Hirschsprung's disease is usually diagnosed in infancy. Although a barium enema can reveal a narrowed aganglionic segment with proximal dilation, which is highly suggestive of Hirschsprung’s disease, the diagnosis is confirmed by biopsy or manometry. It accounts for approximately 20% of cases of partial intestinal obstruction early in infancy. It occurs at a rate of 1 in 5,000 live births and is 4 to 5 times more common in boys. Definitive therapy is surgical resection of the aganglionic segments. Intussusception is the most common cause of intestinal obstruction in children less than 2 years of age, and occurs most commonly in infants between 5 and 12 months of life. There is an estimated incidence of 1 per 2000 children under the age of 15 years with a male predominance. Siblings of affected children have a relative risk 15 to 20 times higher than the general population. Mortality for untreated intussusception is high.

115 307. In an infant with newly diagnosed pyloric stenosis, the classic electrolyte findings of patients with severe symptoms is: a. hyperchloremic, hypokalemic alkalosis. b. hyperkalemic, hypernatremic acidosis. c. hypernatremic, hypokalemic acidosis. d. hypochloremic, hypokalemic alkalosis. e. hypochloremic, hyponatremic acidosis. 308. Which of the following is correct regarding developmental milestones in children? a. 1 month: holds head in midline b. 4 months: rolls c. 6 months: uses pincer response d. 9 months: walks alone e. 12 months: creeps up stairs 309. Sudden infant death syndrome (SIDS): a. is most common in the neonatal period, peaking at 2 months of life, and then tapering off by 4 months. b. is more common in countries where infants sleep supine. c. is primarily a cardiac event. d. does not occur more frequently in infants with apnea of prematurity. e. means a higher risk for similar occurrence in siblings. 310. A clean wound located in a highly vascular area, such as the face, can safely be closed if the patient presents within a maximum of: a. 6 hours. b. 10 hours. c. 18 hours. d. 24 hours. e. 48 hours. 311. A 34-year-old man has extensive lacerations of his left ear after he disagreed with a local bartender over the accuracy of his account. In preparing to reassemble his damaged ear, you tell the patient that you will numb his entire ear by: a. injecting lidocaine without epinephrine and raising a wheal about the entire base of the ear. b. injecting lidocaine with epinephrine directly into the wound. c. injecting lidocaine without epinephrine directly into the wound. d. doing a hemi-facial nerve block. e. performing conscious sedation, as there is no good way to regionally block an ear. 312. The best way to control the pain from a femoral shaft fracture is by using: a. Buck’s traction. b. a femoral nerve block. c. a combination of injectable opioid and non-narcotic pain medicine. d. a steroid injection of the trochanteric bursa. e. high-dose intravenous narcotic injection.

116

307. d th

Rosen 5 , Chapter 165, p. 2297-2301

308. b th

Rosen 5 , Chapter 160, p. 2228

309. d Rosen 5th, Chapter 170, p. 2392-2396

310. d Rosen 5th, Chapter 35, p. 318

311. a th

Rosen 5 , Chapter 35, p. 321

312. b Rosen 5th, Chapter 49, p. 655

Extensive and protracted vomiting in pyloric stenosis may lead to hypokalemia and hyponatremia. More striking decreases occur in chloride concentration and an increase in pH and carbon dioxide content. This constitutes the characteristic changes of hypochloremic alkalosis. A one month old can raise his head only slightly from the prone position. A 2-month old can hold his head in midline and lift his chest off the table. A 4 month old can roll,. At 6-months an infant can sit unsupported, but it is not until 9 months that the pincer grasp begins and the infant starts to cruise and crawl. Walking alone typically occurs at 1 year of age and creeping up stairs occurs at 15 months of age. Apnea of prematurity is not a risk factor for SIDS. SIDS is rare in the first month of life, probably because the neonate has a better anaerobic capacity for survival, and with a gasp may be able to raise his or her arterial PO2 over 20 mmHg and continue breathing. Victims range in age from 1 month to 1 year, with peaks at 2½ months and at 4 months. Epidemiologic studies indicate that the incidence of SIDS is lower in countries where infants sleep supine or in the side-down position. Death is a respiratory rather than cardiac event, and some potential SIDS victims may be successfully resuscitated with ventilation alone. Factors that do not appear to increase the risk of SIDS include maternal alcohol use, recent diphtheria-pertussis-tetanus (DPT) or other vaccine administration, being a sibling of a SIDS victim, sickle cell disease, and bronchopulmonary dysplasia. Although the incidence of bacterial contamination and the risk of infection increase with injuries more than 4 hours old, clean wounds in highly vascular areas (scalp, face, and neck) have an incidence of infection so low that primary closure may be possible even up to 24 hours after injury. Anesthesia of the ear is best achieved by raising a wheal with lidocaine with no epinephrine about the entire base of the ear. This anesthetizes all but the external canal and concha, which, if involved with the injury, require direct infiltration with plain lidocaine. Femoral nerve block is an effective technique for relieving pain of a femoral shaft fracture and is useful in the multitrauma patient when minimizing narcotics is important. The femoral nerve is lateral to the femoral artery at the inguinal ligament and innervates the anterior thigh, the periosteum of the femur, and the knee joint.

117 313. A 74-year-old retired staff surgeon slipped on some urine while visiting his Rosen 5th, Figure wife, who recently had a mastectomy. His right leg is externally rotated and 49-14 shortened, and his x-ray is shown (Figure 49-14). He demands to see the orthopedic surgeon who performed the original procedure, but you tell him that the orthopedic doctor recently moved to Arizona because of the cost of his malpractice insurance. He reluctantly allows you to attempt reduction. You know the best way to get the prosthesis back is through: a. longitudinal traction with simultaneous flexion and internal rotation, then abduction. b. longitudinal traction with simultaneous flexion and external rotation, then adduction. c. longitudinal compression with simultaneous extension and internal rotation, then abduction. d. longitudinal traction with simultaneous extension and external rotation, then abduction. e. longitudinal compression with simultaneous extension and external rotation, then adduction. 314. Herpetic whitlow: a. is usually caused by HSV-2 virus, especially in health-care workers. b. heals more rapidly after incision and drainage. c. is frequently painful and accompanied by axillary adenopathy. d. usually involves several digits simultaneously. e. clears rapidly with topical acyclovir. 315. A 27-year-old woman suffered a deep laceration of her left calf. You cannot see the base of the wound. She tells you “it feels like there’s something in there.” You know that: a. patients who report the sensation of “something in the wound” should be taken seriously, as their perceptions are usually correct. b. ultrasound is the gold standard for finding foreign bodies. c. glass must be leaded for standard radiographs to find. d. plain radiographs find less than 50% of foreign bodies. e. it is not essential to find foreign bodies, as they inevitably “work their way out.” 316. The drug of choice to treat Rocky Mountain Spotted Fever in a 6 year-old girl is: a. azithromycin. b. chloramphenicol. c. doxycycline. d. cephalexin. e. linezolid. 317. Most cases of infant botulism are caused by ingestion of: a. well-water. b. canned baby food. c. inadequately prepared instant formula. d. raw chicken. e. raw honey.

118

313. a th

Rosen 5 , Chapter 49, p. 665-666

314. c th

Rosen 5 , Chapter 43, p. 530-531

315. a Rosen 5th, Chapter 53, p. 770-771

316. b Rosen 5th, Chapter 128, p. 1895

317. e Rosen 5th, Chapter 123, p. 1795

Treatment for the dislocation is early closed reduction, usually under conscious sedation. Strong, in-line traction is done with simultaneous flexion and internal rotation. Finally, the hip is abducted once the head clears the rim of the acetabulum. Herpetic whitlow is a primary or recurrent HSV infection of the finger. HSV-1 is seen in health care workers who are exposed to infected oral secretions. HSV-2 is more common among adults due to digital/genital contact in the community. The disease is usually limited to a single digit. Herpetic whitlow is frequently painful and accompanied by axillary adenopathy. Vesicles, which may be recognizable early in the course of the disease, coalesce and may appear to contain pus, but actually contain necrotic epithelial cells causing the purulent appearance. Whitlow may be misdiagnosed as a paronychia and incised, which may delay healing or allow a secondary infection to occur. For patients with frequent painful recurrences suppressive therapy with systemic acyclovir may be effective. Topical acyclovir has not been shown to be effective in either the treatment or prophylaxis of this disorder. If a foreign body is suspected but not found during exploration of a wound, a plain film should be ordered first, since plain radiography will detect as many as 80 to 90% of all foreign bodies. When a foreign body is suspected but cannot be palpated or visualized, plain films should be performed in an attempt to determine the location, size, and number of foreign objects present. Even when the foreign body is not visualized, radiographs may show secondary changes (e.g., pulmonary air trapping) providing clues to foreign body presence. To assist in the localization of the object, two views— anteroposterior (AP) and lateral—are usually necessary. Objects such as metal or gravel, which are denser than the tissue in which they are embedded, are usually easy to visualize on plain radiographs. However, organic material, which has a density similar to that of human tissue, may not be seen on plain films. Besides plain radiographs, other techniques may be useful for ED workup of foreign bodies. These modalities include fluoroscopy, contrast studies, computed tomography (CT) scanning, magnetic resonance imaging (MRI), and ultrasound (US). Tetracycline or doxycycline is the antibiotic of choice to treat Rocky Mountain Spotted Fever, but chloramphenicol can be used for patients allergic to tetracycline and in children younger than 9 years of age. A usual course is 6 to 10 days and should continue for 72 hours after defervescence. Sulfa drugs should be avoided, as they can exacerbate the illness. Rickettsiae are routinely resistant to penicillins, cephalosporins, aminoglycosides, and erythromycin. Adults can get botulism not by bacterial infestation but by ingestion of botulinum toxin through exposure to foods such as home-canned vegetables. Unlike adults, infants, whose guts are not colonized fully, are susceptible to infection with viable bacteria that elaborate toxin. Parents of infants should be questioned for possible ingestion of C. botulinum spores, commonly transmitted by feeding honey, but occasionally reported with corn syrup.

119 318. Synchronized intermittent ventilation (SIMV) prevents: a. decreased renal blood flow. b. decreased venous return to the heart and decreased cardiac output. c. nosocomial infections of the lungs and sinuses. d. respiratory alkalosis. e. stacking breaths, which can lead to barotrauma. 319. A majority of acute hemolytic transfusion reactions result from: a. antibodies in the donor's type O serum against the recipient's A or B antigen. b. antigen-antibody reaction to the Rh factor. c. emergent use of uncrossed matched type O-negative blood. d. transfusion of the wrong unit of blood due to clerical error. e. undetected leukocyte antibodies. 320. The safest place in the body for hydrocarbons is the: a. stomach. b. colon. c. duodenum. d. jejunum. e. ileum. 321. Rhabdomyolysis is defined by: a. end organ damage. b. total serum creatine kinase more than five times normal. c. total serum creatine kinase higher than 1000. d. total serum myoglobin more than three times normal. e. total plasma myoglobin higher than 1 mg/dl. 322. Eikenella corrodens is a worrisome bacteria found most commonly in injuries caused by: a. cat bites. b. foot punctures through an old sneaker. c. lionfish spines. d. human bites. e. camel bites. 323. Twenty-four hours after she was exposed to an unknown amount of radiation, a 45-year-old woman has an absolute lymphocyte count of 1500. This means she: a. will almost certainly die, no matter what treatment you provide. b. has a 50:50 chance of survival. c. can survive, but only after aggressive therapy. d. had exposure in excess of 2000 rad. e. requires no special care, as she is expected to survive without complications. 324. An example of an ester local anesthetic is: a. lidocaine. b. tetracaine. c. mepivacaine. d. bupivacaine. e. prilocaine.

120

318. e th

Rosen 5 , Chapter 2, p. 22

319. d th

Rosen 5 , Chapter 5, p. 50

320. a Rosen 5th, Chapter 152, p. 2162

321. b th

Rosen 5 , Chapter 121, p. 1766

322. d th

Rosen 5 , Chapter 54, p. 781

323. e Rosen 5th, Chapter 140, p. 2059-2060

324. b Rosen 5th, Chapter 182 p.2571

Synchronized intermittent ventilation (SIMV) supports a patient's spontaneous breath at a preset rate that prevents a mechanical breath being delivered at the same time as a spontaneous breath (stacking). The other choices are all potential adverse effects of positive-pressure ventilation. Acute intravascular hemolytic transfusion reactions are immediate and life threatening. They are usually the result of ABO incompatibility, most often due to clerical error (e.g., mislabeling of blood product, misidentification of patient). Because RBCs of type O do not have A or B antigens on their surface, they are not agglutinated or hemolyzed by anti-A or anti-B antibodies. Less severe and more common febrile transfusion reactions are due to antileukocyte and antiplatelet antibodies, seen in multiply transfused patients. The aphorism, “the safest place in the body for hydrocarbons is the duodenum,” holds true for most hydrocarbons regardless of the volume ingested. There is a high toxicity to the lungs, but low systemic toxicity to the gastrointestinal track, with low potential for GI absorption. A specific creatine kinase level cannot define rhabdomyolysis. In general, however, a creatine kinase level greater than five times normal is diagnostic, but levels as high as several hundred thousand have been reported. Eikenella corrodens, a facultatively anaerobic gram-negative rod harbored in human dental plaque, is found in 25% to 29% of human closed fist injury infections. It acts synergistically with aerobic organisms, most frequently streptococci, and is thought to account for greater morbidity in these wounds. E. corrodens is susceptible to penicillin, ampicillin, second- and third-generation cephalosporins, carbenicillin, tetracycline, and the fluoroquinolones. The lymphocyte count 24 hours postexposure is useful in predicting the patient’s clinical course. If the lymphocyte count is maintained above 1200/mL, it is unlikely the patient has received a lethal dose of radiation and no clinical support is required. If the absolute lymphocyte count falls between 300 and 1200 at 48 hours, the possibility of exposure to a lethal dose of radiation should be suspected. Levels less than 300 are critical and require heroic measures. Local anesthetic agents are chemical compounds that consist of an aromatic and an amine group separated by an intermediate chain. The class that has an ester link between the intermediate chain and aromatic portion are called amino esters; procaine, chloroprocaine, and tetracaine are the only ones in use today. Amides have an amide link and are more commonly used; lidocaine, mepivacaine, prilocaine, bupivacaine, and etidocaine are examples (Notice that they all have two ‘i’s). Esters are unstable in solution and are metabolized in the body by the plasma enzyme cholinesterase. Enzymes in the liver destroy the amides after absorption.

121 325. The LD50 from exposure to ionizing radiation is defined as the dose of penetrating ionizing radiation that will result in the deaths of 50% of the exposed population within 60 days. In humans who receive aggressive post-exposure therapy, this number is generally accepted to be: a. 0.5 Gy (50 rad). b. 1.0 Gy (100 rad) c. 4.5 Gy (450 rad). d. 6.0 Gy (600 rad) e. 10.0 Gy (1000 rad). 326. Hyperbaric oxygen therapy is contraindicated in the patient with: a. acute sinusitis. b. pregnancy. c. disulfiram (Antabuse®) therapy. d. epilepsy. e. metallic prostheses. 327. Chondromalacia patellae: a. is a traumatic syndrome of the patella ligament. b. is caused by patellofemoral malalignment, leading to excessive lateral pressure on the articular cartilage. c. is most common in adolescent male athletes d. causes joint effusion and generalized knee pain. e. is made worse with descending stairs and squatting down. Rosen 5th, Figure 50-9

328. Several young men coveted the jacket previously worn by the patient whose x-ray is shown here (Figure 50-9). He refused to relinquish this valued possession, having worked long and hard at a menial position in order to earn the money to honestly purchase it. After forcefully removing the aforementioned jacket, the assailants jumped up and down on his leg a few times. The nerve most likely to be affected is the: a. peroneal. b. superficial femoral. c. deep femoral. d. tibial. e. sciatic. 329. A normal reticulocyte count is: a. 0.5 – 1.5% b. 2.0 – 5.0% c. 6.0 – 7.5% d. 7.5 – 10.0% e. >10%

122

325. c th

Rosen 5 , Chapter 140, p. 2059

326. c th

Rosen 5 , Chapter 194, p. 2700

327. b Rosen 5th, Chapter 50, p. 688-689

328. a Rosen 5th, Chapter 50, p. 689

329. a Rosen 5th, Chapter 115, p. 1665

The median lethal whole body dose (i.e., the dose that is lethal for 50% of test subjects)for human beings is estimated to be about 4.5 Gy if victims are hospitalized and receive aggressive care. With whole-body exposure of more than 10 Gy, mortality is nearly 100% unless bone marrow transplantation is performed. Very few absolute contraindications to HBO therapy exist. Treatment with doxorubicin, cisplatin, bleomycin, disulfiram, and mafenide acetate is considered a contraindication to HBO therapy because of potentially toxic effects when combined with HBO. An untreated pneumothorax is an absolute contraindication because of the concern that it may progress to a tension pneumothorax, particularly during decompression. The patellofemoral pain syndrome refers to the clinical presentation of anterior knee pain related to changes in the patellofemoral articulation. The term chondromalacia has been used imprecisely to define this syndrome, but this is a pathologic term that refers to softening of the articular cartilage. A typical presentation is nonspecific anterior knee discomfort that is nonradiating and occurs in a teenage girl. The knee is more painful with prolonged flexion, and the discomfort is typically accentuated by stair climbing and kneeling. The popliteal neurovascular bundle, composed of the popliteal artery, popliteal vein, and common peroneal nerve, runs posteriorly behind all bony and ligamentous structures in the popliteal fossa. The popliteal artery is fixed in the fibrous tunnel of the adductor magnus hiatus proximally and traverses the fibrous arch of the soleus and interosseous membrane distally. In essence, it is tethered to both the femur and tibia, and its inherent immobility renders it susceptible to injury during dislocation. At that time, the popliteal artery may be stretched, lacerated, or contused. Because of the parallel course of the popliteal vein and peroneal nerve, they are vulnerable to a similar injury. When the late normoblast extrudes its nucleus, it still contains a ribosomal network, which identifies the reticulocyte. The reticulocyte retains its ribosomal network for about 4 days, of which 3 are spent in the bone marrow and 1 in the peripheral circulation. The red blood cell matures as the reticulocyte loses its ribosomal network and circulates for 110 to 120 days. The erythrocyte is then removed by macrophages that detect senescent signals. Thus at any given time, approximately 1% of circulating erythrocytes can be identified as reticulocytes.

123 330. A 56-year-old man was found unconscious in a nearby wooded area. Medics report that his dog was lying dead next to him. As you undress him for examination, you see these skin findings and realize the cause of his coma is exposure to: a. cyanide. b. hydrofluoric acid. c. herbicide aerial spray. d. lightning. e. carbon tetrachloride. 331. Metformin: a. is more appropriately termed an antihyperglycemic than a hypoglycemic agent. b. decreases peripheral sensitivity to insulin. c. suppresses exogenous glucose uptake. d. stimulates insulin secretion. e. causes profound hypoglycemia if taken in error by a non-diabetic. 332. A 63-year-old woman developed acute left-sided weakness and garbled speech about 40 minutes prior to arrival. Her head CT is unremarkable and she has no contraindications, so you choose to treat her with: a. low-molecular weight heparin, 1 unit/kg subcutaneously. b. recombinant tissue plasminogen activator, 0.9 mg/kg, maximum 90 mg; first 10% administered as bolus, with remaining amount infused over 60 minutes. c. recombinant tissue plasminogen activator 90 mg IV bolus. d. streptokinase IV. e. nothing has been shown effective in this scenario. 333. A comatose 19-year-old man is brought from his college dormitory room. His roommate found him at the end of classes today, but did not accompany him to the hospital. Fortunately he is wearing a Medic Alert bracelet that identifies him as diabetic. He also smells like fingernail polish remover. His bedside glucose is too high to be read, so you empirically start an intravenous with normal saline solution after blood work is sent to the laboratory. You get a call several minutes later with some “panic values:” Na+: 129, K+: 3.8; HCO3- : 12, glucose: 745. You know that the corrected sodium is actually: a. 119 b. 129 c. 134 d. 139 e. 145 334. A 47-year-old diabetic woman complains of a severe headache, fever, and a green-black nasal discharge. She has generalized left facial swelling and redness, but does not have proptosis. You see necrosis of the inferior turbinate, and a black spot on the soft palate. You know that she will require aggressive treatment with intravenous: a. vancomycin. b. gentamicin. c. cyclosporine. d. dexamethasone. e. amphotericin B.

124

330. d th

Rosen 5 , Chapter 136, p. 2014

331. a th

Rosen 5 , Chapter 120, p. 1759

332. b Rosen 5th, Chapter 95, p. 1440-1441

333. d Rosen 5th, Chapter 119, p. 1724

334. e Rosen 5th, Chapter 70, p. 982, 983

Feathering burns are not true burns because there is no damage to the skin itself. They seem to be caused by electron showers induced by the lightning that make a fern pattern on the skin. These fern patterns are called Lichtenberg figures and are pathognomonic for lightning. Metformin works by decreasing hepatic glucose output, leading to decreased insulin resistance and lower blood glucose. Used alone, metformin does not cause hypoglycemia; it is contraindicated in patients with renal insufficiency and metabolic acidosis, and must be used with caution in patients with hypoxemia, liver compromise, and alcohol abuse. To date, the Food and Drug Administration (FDA) have approved only the use of intravenous tPA for treatment of patients with acute ischemic stroke. These recommendations were based on the results of the National Institutes of Neurological Disorders and Stroke (NINDS) trial. The total dose of rt-PA is 0.9 mg/kg, with a maximum dose of 90 mg; 10% of the dose is administered as a bolus, with the remaining amount infused over 60 minutes. Heparin is frequently prescribed for patients with acute ischemic stroke or TIAs, but its value is unproved. Streptokinase is not recommended for use in patients with acute ischemic strokes. 745 – 100 = 645 / 1.6 = 10.3 + 129 = 139. Redistributive hyponatremia is caused by osmotically active solutes in the extracellular space that draw water from the cell, diluting the serum sodium concentration. Common situations causing such hyperosmolar states include hyperglycemia (e.g., diabetic ketoacidosis) and parenteral administration of mannitol or glycerol for the management of intracranial hypertension or glaucoma. The measured serum sodium in patients with hyperglycemia can be corrected by adding approximately 1.6 mEq/L for every 100-mg/dl rise in the serum glucose over 100 mg/dl. Invasive fungal sinusitis is an aggressive opportunistic rhinocerebral infection that affects immunocompromised hosts. Mucormycosis (Rhizopus) is usually associated with fever, localized nasal pain, and cloudy rhinorrhea. On examination, the affected tissue (usually the turbinates) appears gray, friable, anesthetic, and nonbleeding because of infarction caused by mucormycotic angioinvasion. In advanced cases the tissues are necrotic and black, and the infection spreads beyond the sinus. Acute fulminant fungal sinusitis requires emergent consultation and admission for IV antifungal therapy and aggressive surgical debridement.

125

335. This 56-year-old woman was started on a new medication last week after she visited a walk-in clinic. Today she is toxic in appearance, and large sheets of skin are peeling from her back (Figure 114-8). The old chart is not available, and the patient only remembers that her “face hurt.” You recognize that the most likely culprit is: a. prednisone for Bell’s palsy. b. doxycycline for Lyme-induced Bell’s palsy. c. clindamycin for dental abscess. d. carbamazepine for trigeminal neuralgia. e. cephalexin for erysipelas.

Rosen 5th, Figure 114-8

126

335. d th

Rosen 5 , Chapter 114, p. 1643

The main feature of nonstaphylococcal-induced toxic epidermal necrolysis (TEN), or Lyell’s disease, is the separation of large sheets of epidermis from underlying dermis. The full thickness of epidermis is involved. The two conditions are easily histologically distinguishable with a skin biopsy. A mortality rate of 15% to 20% is expected with this condition. Drugs including the long-acting sulfa drugs, penicillin, aspirin, barbiturates, phenytoin, carbamazepine, allopurinol, and nonsteroidal antiinflammatory drugs are an important cause of TEN.

127

Abdominal and Gastrointestinal Disorders 336. In adults the most common source of upper gastrointestinal bleeding is: a. Mallory-Weiss tear. b. peptic ulcer disease. c. esophageal varices. d. esophagitis. e. gastric erosions. 337. Melana is: a. present in 95% of upper gastrointestinal bleeds. b. from blood that has been in the GI tract for at least 24 hours. c. present in approximately one-third of lower gastrointestinal bleeds. d. may occur from as little as 10 mls of blood in the GI tract. e. stool that will remain black and tarry for no more than 24 hours after bleeding stops. 338. Antiemetic drugs work by: a. suppressing the vomiting center in the brain. b. depressing the chemoreceptor trigger zone. c. inhibiting impulses to the chemoreceptor trigger zone from the vestibular apparatus. d. inhibiting impulses to the vomiting center from peripheral receptors. e. all of the above. 339. In adults the most common cause of nausea and vomiting is: a. cholecystitis. b. medication. c. kidney stones. d. migraine headache. e. myocardial infarction. 340. A patient is sent to the emergency department by his primary doctor to rule out gastric outlet obstruction. After talking to him, you also believe that gastric outlet obstruction is the most likely diagnosis given his symptom of: a. emesis that is both bilious and bloody. b. abdominal distention with normoactive bowel sounds. c. vomiting during meals. d. vomiting made worse by lying supine. e. vomiting of material eaten more than 12 hours previously. 341. The most common cause of neuromuscular swallowing dysfunction is: a. stroke. b. myasthenia gravis. c. poliomyelitis. d. polymyositis. e. tetanus.

128

336. b th

Rosen 5 , Chapter 23, p. 194-195

337. c th

Rosen 5 , Chapter 23, p. 195

338. e Rosen 5th, Chapter 21, p. 179

339. b Rosen 5th, Chapter 21 p. 178

340. e th

Rosen 5 , Chapter 21, p. 180-181

341. a th

Rosen 5 , Chapter 84, p. 1245

In adults the most common causes – in descending order – of upper gastrointestinal bleed are: peptic ulcer disease , gastric erosions, varices, and Mallory-Weiss tears. The top three account for approximately three fourths of adult patients with upper GI bleed.. Melena, or black tarry stool, occurs from approximately 150-200 ml of blood in the GI tract. Melena is present in approximately 70% of upper GI bleeds and 33% of lower GI bleeds. Black nontarry stool may result from as little as 60ml of blood. Blood from the jejunum or duodenum must remain in the GI tract for at least 8 hours before turning black. Stool may remain black and tarry for several days after bleeding has stopped. The chemoreceptor trigger zone area is rich in dopamine D2 receptors, which are antagonized by drugs such as prochlorperazine, metoclopramide, and droperidol. The serotonin receptor has been found widely in the area postrema and the GI tract. It may act both directly and through the release of dopamine. Serotonin receptor antagonists (ondansetron, alosetron, granisetron) have been shown to be effective in preventing chemotherapy-induced nausea and vomiting. Concentrations of cholinergic and histamine receptors are found in the lateral vestibular nucleus and are important in motion sickness. Meclizine, diphenhydramine, and scopolamine act by antagonizing these receptors. In adult medicine, nausea and vomiting are most often caused by medications. The most common causes of nausea and vomiting are acute gastroenteritis, febrile systemic illnesses, and drug effects. Acute viral gastroenteritis is the most common GI disease in the United States. Vomiting of material eaten more than 12 hours previously is pathognomonic for outlet obstruction. Bilious emesis essentially rules out a gastric outlet obstruction. Highpitched bowel sounds suggest a gastric outlet obstruction. Neuromuscular diseases cause approximately 80 % of oropharyngeal dysphagias. Strokes are probably the most common cause of neuromuscular dysphagia, and pharyngeal muscle weakness is often the mechanism. The second most common cause of neuromuscular dysphagia is polymyositis or dermatomyositis. These disorders are characterized by inflammatory and degenerative changes that produce weakness of the palate, pharynx, and upper esophagus.

129

342. A 25-year-old man complains of a severe intermittent substernal chest pain. It started suddenly while he was lying in bed. He admits to several months of difficulty swallowing solids and liquids, and he occasionally has to stand when he eats to help food pass into his stomach. Physical examination reveals an underweight male with no abnormal findings. His 12-lead ECG is normal. His probable diagnosis is: a. gastroesophageal reflux. b. achalasia. c. nutcracker esophagus. d. spontaneous esophageal perforation (Boerhaave’s syndrome). e. duodenal ulcer. 343. In distinguishing chest pain caused by gastroesophageal disease from that caused by coronary disease, the most helpful historical point is probably: a. pain made worse by postural changes and swallowing. b. pain precipitated by high emotional state. c. pain which is almost always relieved by antacids. d. pain precipitated by exercise and relieved by rest. e. radiation of pain into the neck or the jaw. 344. The majority of esophageal perforations are: a. spontaneous. b. iatrogenic. c. traumatic. d. caused by caustic ingestion. e. idiopathic. 345. One of the most common etiologies of peptic ulcer disease is: a. dietary indiscretion. b. Helicobacter pylori infection. c. idiopathic. d. increased parietal cell mass. e. Zollinger-Ellison syndrome. 346. The description of abdominal pain may be helpful in its evaluation. For instance: a. pain from ulcer disease is often described as colicky. b. acute gastric volvulus is likely to present with mild pain. c. peptic ulcer pain is classically described as a burning or gnawing quality. d. pain from iatrogenic esophageal perforation always occurs immediately after the procedure. e. pain from a perforated duodenal ulcer is usually appreciated in the epigastrium only and rarely becomes generalized. 347. Treatment of choice for acute gastric volvulus is: a. watchful waiting. b. endoscopic decompression. c. combined Trendelenberg / left lateral decubitus position. d. attempted reduction by barium swallow. e. surgery.

130

342. b th

Rosen 5 , Chapter 84, p. 1246

343. a Rosen 5th, Chapter 84, p. 1238, 1240

344. b Rosen 5th, Chapter 38, p. 406-409 Chapter 84, p. 1236

345. b th

Rosen 5 , Chapter 84, p. 1241

346. c th

Rosen 5 , Chapter 84, p. 1236, 1242-1244

347. e Rosen 5th, Chapter 84, p. 1244

Achalasia is a disorder in which there is a marked increase in the resting pressure of the lower esophageal sphincter and absent peristalsis in the body of the esophagus. Dysphagia is common. Odynophagia, from esophageal spasm may be seen early in the course of the disease. Symptoms often worsen with rapid eating and stress. A dilated esophagus with air-fluid levels may be seen on chest x-ray. Esophageal pain is more likely to be positional and related to swallowing. Radiation of pain is an inconstant finding in both esophageal and cardiac chest pain. Precipitation of pain by exercise and relief by rest may occur in pain from reflux or ischemic heart disease. Emotional precipitation of pain occurs in reflux, although it is also seen in coronary artery disease. Relief of chest pain from reflux by antacids is a key point in the history, however the emergency physician should not place too much weight upon this point as evidence against a cardiac etiology. The relief is often short lived, and may recur in a short time. Spontaneous esophageal perforation accounts for only 15% of cases, with iatrogenic injuries accounting for most of the remainder. These usually occur as a complication of upper endoscopy, dilation, sclerotherapy, or other GI procedures.

Peptic ulcer disease is considered to have two main etiologies: Helicobacter pylori infection and NSAID use. Only about 1% of PUD is caused by increased levels of circulating gastrin from gastrin-secreting tumors (Zollinger-Ellison syndrome). These patients have increased parietal cell mass and hypersecretion of acid. Colicky pain is rarely gastric or duodenal in origin. Classically ulcer pain is described as nonradiating epigastric pain of a burning or gnawing quality. Primary volvulus may present with the sudden onset of severe pain. Pain from an iatrogenic perforation may not appear until several hours after the procedure. Pain from a perforated duodenal ulcer is usually appreciated first in the epigastrium but becomes generalized within a short time Treatment of an acute gastric volvulus is surgical. Acutely, one should attempt passage of a nasogastric tube, as this may occasionally reduce the volvulus. There have also been reports of reductions of volvulus using endoscopy, but this is best reserved for patients who cannot tolerate surgery or who have no evidence of vascular compromise.

131 348. A patient with a known exposure to hepatitis B presents with right upper quadrant abdominal pain and scleral icterus. You know that her serum bilirubin must be at least: a. 1.5 mg/dl. b. 2.0 mg/dl. c. 2.5 mg/dl. d. 3.0 mg/dl. e. 3.5 mg dl. 349. Of the tests listed, the one MOST ACCURATE in predicting a fulminant course of disease in a patient with hepatitis is: a. prolonged prothrombin time. b. bilirubin higher than 10 mg/dl. c. AST several times higher than ALT. d. alkaline phosphatase more than 5 times normal. e. lactate dehydrogenase more than 10 times normal. 350. A newly hired housekeeper is stuck with a hollow needle while picking up trash in a room recently vacated by a patient known to have chronic hepatitis B. She has not yet started her primary hepatitis prophylaxis series. You decide to: a. send antibody levels (HBSAg) on the employee and determine treatment based on the results. b. give her hepatitis B immune globulin (HBIG) 0.06 ml/kg and start the hepatitis B vaccine series, with the first shot in the buttock. c. give her hepatitis B immune globulin (HBIG) 0.06 ml/kg and start the hepatitis B vaccine series, with the first shot in the deltoid. d. give her hepatitis B immune globulin (HBIG) 0.6 ml/kg and start the hepatitis B vaccine series, with the first shot in the buttock. e. give her hepatitis B immune globulin (HBIG) 0.6 ml/kg and start the hepatitis B vaccine series, with the first shot in the deltoid. 351. Ascites occurs as a consequence of: a. portal hypertension. b. impaired hepatic lymph flow. c. hypoalbuminemia. d. renal salt retention. e. all of the above. 352. The organism which causes the majority of spontaneous bacterial peritonitis is: a. Streptococcus pneumoniae. b. Escherichia coli. c. Klebsiella pneumoniae. d. Staphylococcus aureus. e. Pseudomonas abdominus 353. In children, the majority of acute esophageal obstructions are caused by: a. balloons. b. coins. c. food. d. strictures. e. surgical procedures.

132

348. c th

Rosen 5 , Chapter 85, p. 1253

349. a th

Rosen 5 , Chapter 85, p. 1253

350. c Rosen 5th, Chapter 85, p. 1256

351. e Rosen 5th, Chapter 85, p. 1259

352.b Rosen 5th, Chapter 85, p, 1261

353. b Rosen 5th, Chapter 84, p. 1234

Scleral icterus usually is not clinically apparent, even to the most astute observers, until serum bilirubin is above 2.5 mg/dl. Muddy sclera, commonly found among African-American patients, may obscure or confuse this finding. An alternative in this setting is examination of sublingual or subungual surfaces. Prothrombin time (PT) is a useful test to assess the degree of hepatic synthetic dysfunction. Elevation of the PT may be the first clue of a complicated course. The white blood count (WBC) generally is not useful because values range from low overall counts with a lymphocytic predominance to marked polymorphonuclear leukocytosis. The typical case is associated with elevations (tenfold to 100-fold) of serum aspartate aminotransferase (AST) and alanine aminotransferase (ALT), with ALT generally elevated in excess of AST. Bilirubin may be moderately increased (5 to 10 mg/dl) and occasionally is markedly elevated (15 to 25 mg/dl). Alkaline phosphatase and lactate dehydrogenase may be elevated but are rarely more than two to three times normal. HBV immune globulin (HBIG) is recommended for immediate passive immunization in individuals not previously immunized but exposed to potentially infective material. HBIG alone diminishes the risk of HBV infection by 75%. Unvaccinated, exposed people should receive HBIG 0.06 ml/kg intramuscularly (IM) in addition to the HB vaccine. Centers for Disease Control and Prevention (CDC) data suggest that optimal immunologic response results from deltoid injection. Ascites occurs as a consequence of portal hypertension, impaired hepatic lymph flow, hypoalbuminemia, and renal salt retention. Although ascites generally causes little more than unsightly abdominal distention and discomfort, it can become massive and lead to respiratory compromise. E. coli account for approximately 47% to 55% of spontaneous bacterial peritonitis infections, Streptococcus sp 18%-26%, Klebsilla sp 11%, and Streptococcus pneumoniae 8%-26%. Polymicrobial and anaerobic infections have been reported but are not common. A third-generation cephalosporin such as cefotaxime is considered to be a drug of choice with a demonstrated cure rate of 85%. An alternative would be an ampicillin sulbactam combination. Ampicillin and an aminoglycoside is a rational and effective combination but somewhat less desirable due to the risk of renal toxicity. Eighty percent of foreign body ingestions occur in the pediatric age group, with coins being the most commonly impacted object. In adults, most impactions are due to pieces of food, particularly meat and bones.

133 354. Drug-induced liver injury usually causes damage by hepatocellular necrosis or cholestasis. A commonly used drug which causes hepatonecrotic changes is: a. amiodarone. b. anabolic steroids. c. chlorpromazine. d. erythromycin estolate. e. haloperidol. 355. The most likely cause of massive lower GI bleed in a child under two years of age is : a. duodenal atresia. b. gastric ulcer. c. Mallory-Weiss tears. d. Meckel’s diverticulum. e. variceal bleeding secondary to portal hypertension. 356. The most common cause of massive lower GI bleed in adults is: a. diverticular disease. b. inflammatory bowel disease. c. internal hemorrhoids. d. left colon carcinoma. e. right colon carcinoma. 357. Post-transfusion hepatitis is usually due to: a. hepatitis A. b. hepatitis B. c. hepatitis C. d. hepatitis D. e. hepatitis E. 358. An ill-appearing, 47-year-old man complains of severe epigastric pain and vomiting. Lab studies show a markedly elevated lipase. He is not a drinker. The cause is probably: a. biliary tract disease. b. cimetidine therapy. c. hyperlipidemia. d. indomethacin therapy. e. infection. 359. The second most common cause of small bowel obstruction in the United States: a. abscess. b. congenital abnormality. c. Crohn’s disease. d. hernia. e. regional enteritis. 360. Epigastric pain which is relieved by food, but which returns between meals is typical for: a. sliding hiatal hernia. b. esophagitis. c. peptic ulcer. d. pancreatitis. e. Zollinger-Ellison syndrome.

134

353. a th

Rosen 5 , Chapter 85, p. 1261-1262

355. d Rosen 5th, Chapter 23, p. 194-195

356. a Rosen 5th, Chapter 23, p. 194-195

357. c th

Rosen 5 , Chapter 85, p. 1252

358. a Rosen 5th, Chapter 86, p. 1273

359. d Rosen 5th, Chapter 87, p. 1284

360. c Rosen 5th, Chapter 84, p. 1242

Although specific agents tend to cause damage characterized by a particular pattern of injury, there is considerable overlap. Cellular necrosis is commonly associated with anesthetic agents (e.g., halothane), the antifungals amphotericin and ketoconazole, or the antidysrhythmic amiodarone. A cholestatic picture is characteristic of chlorpromazine, haloperidol, anabolic or oral contraceptive steroids, and erythromycin estolate. Esophagitis, gastritis, peptic ulcer disease, esophageal varices, and Mallory-Weiss tears are common causes of upper GI bleeding in children . In children under the age of 2 years, massive LGIB is most often a result of Meckel’s diverticulum or intussusception. Anal fissure, infectious colitis, inflammatory bowel disease, and polyps are other causes of LGIB in children. At all ages rectal abnormalities are the most common cause of minor LGIB The most common etiology of significant LGIB in adults is diverticulosis followed by angiodysplasia . Although hepatitis C has been most prominently associated with transfusions, only 10% of patients with this disease report a previous history of having received blood or blood products. The historic risk of hepatitis in patients receiving blood transfusions was about 0.45% per unit transfused. The screening of donor blood for surrogate markers (aminotransferases) and antibody to hepatitis C has decreased this risk to 0.03% per unit. Eighty percent of pancreatitis is caused by either gallstones (about 45%) or alcoholism (about 35%). The exact mechanism of biliary pancreatitis is not clear. Either a stone within the bile duct applies transmural pressure on the pancreatic duct or a stone in the common channel of the pancreatic duct and common bile duct causes obstruction. Obstruction or pressure on the pancreatic duct causes bile reflux or increased pressure of pancreatic secretions. Either mechanism leads to the activation of pancreatic enzymes setting off the cascade of pancreatitis. Many cases that were presumed to be idiopathic are actually due to small stones, sludge, or crystals that are too small to be seen by ultrasound examination but may be noted on endoscopic retrograde cholangiopancreatography (ERCP). In developed countries, postoperative adhesions are responsible for more than 50% of all small bowel obstruction. It is estimated that as many as 15% of abdominal surgeries will eventually result in small bowel obstruction from adhesion. A particularly high incidence of small bowel obstruction is found after gynecologic or intestinal surgeries, as well as in those patients who have previously undergone surgery in the presence of peritonitis or significant abdominal trauma. Other important causes of small bowel obstruction include hernias and neoplasms, each with an incidence of approximately 15%. Hernias can be either external or internal. The most common symptom of PUD is abdominal pain, occurring in 94% of patients with an ulcer. Classically, ulcer pain is described as nonradiating epigastric pain of a burning or gnawing quality, although pain in the chest, back, or other areas of the abdomen may be noted. Relief of pain after eating is a feature of gastric or duodenal ulcer. Pain usually occurs 2 to 3 hours following a meal or at night.

135 361. You are evaluating a 57-year-old woman with epigastric discomfort, and suspect pancreatitis. You know that: a. endocrine products produced by the pancreas include amylase, lipase, trypsin, and other enzymes. b. in patients with chronic pancreatitis, the degree of pain diminishes with the decreasing amount of remaining pancreatic tissue. c. the bile duct, portal vein, splenic vein, vena cava, aorta, and superior mesenteric artery all lie posterior to the pancreas. d. pancreatic tumors are usually painful and rapidly progressive. e. the most common pancreatic disorder is pancreatitis. 362. The pain of uncomplicated small bowel obstruction is frequently: a. a poorly localized, crampy abdominal pain. b. a constant and severe abdominal pain. c. associated with peritoneal signs. d. mild in proximal intestinal obstructions. e. associated with explosive profuse diarrhea. 363. A 6-year-old boy is brought by his frantic mother because she found blood on the toilet paper. You know that the most common cause of this complaint in this age group is: a. anal fissure. b. cryptitis. c. external hemorrhoid. d. internal hemorrhoid. e. perirectal abscess. 364. Which of these conditions may be drained appropriately in the emergency department? a. “horseshoe” abscess. b. intersphincteric abscess. c. postanal abscess. d. perianal abscess. e. supralevator abscess. 365. Painful anorectal vesicles are most commonly caused by: a. Chlamydia trachomatis. b. Herpes simplex. c. Neisseria gonorrhea. d. papillomavirus. e. Treponema pallidum. 366. The most prevalent parasite in the United States is: a. Cyclospora cayetanensis. b. Entamoeba histolytica. c. Enterobius vermicularis. d. Giardia lamblia. e. Plasmodium vivax.

136

361. c th

Rosen 5 , Chapter 84, p. 1272, 1281

362. a Rosen 5th, Chapter 87, p. 1285

363. a Rosen 5th, Chapter 23, p. 195 Chapter 91, p.1347

364. d th

Rosen 5 , Chapter 9, p. 1348-1349

365. b th

Rosen 5 , Chapter 91, p. 1353-1354

366. c th

Rosen 5 , Chapter 89, p. 1318

The pancreas has both essential exocrine and endocrine functions. Exocrine products include amylase, lipase, trypsin, chymotrypsin, elastase, carboxypeptidase, phospholipase, and other enzymes. The endocrine functions of the pancreas are managed by insulin, glucagon, pancreatic polypeptide, and somatostatin. Advances in care have decreased hospital mortality from pancreatitis from 25% to 30% to as low as 6% to 10% over the past 30 years. Anterior to the pancreas from right to left is the transverse colon, the lesser sac of the omentum, and the stomach. Posteriorly lies the bile duct, portal vein, splenic vein, vena cava, aorta, and superior mesenteric artery. The presentation of pancreatic cancer is variable because progression of the disease is indolent. Diabetes is the most common disorder of the pancreas, followed by pancreatitis. Patients with small bowel obstruction typically complain of regularly recurrent bouts of poorly localized abdominal pain lasting from seconds to minutes. Several hours of severe colicky pain in association with bilious vomiting and mild abdominal distension is typical of proximal intestinal obstruction, whereas a day or two of progressively worsening pain and more prominent abdominal distention is typical of distal obstruction. A change in the description of the pain from intermittent and colicky to constant and severe may signal the development of complications, such as intestinal ischemia or perforation. The presence of peritoneal signs usually indicates late obstruction with complications including strangulation. Patients with complete obstruction eventually develop obstipation, while those with early or partial obstruction may continue to pass stool or flatus. The most common cause of lower GI bleeding in children is an anal fissure. It is also the most common anorectal problem in the pediatric patient.

Perirectal and perianal abscesses are the most common (40% - 45%) and can be incised and drained in the Emergency Department in the absence of complicating factors.

In herpes proctitis single or coalesced vesicles and ulcerations occur in the perianal area and rectum. Syphilis (Treponema pallidum) may form a perianal ulcer or chancre. Chlamydia may form a perianal ulceration .Gonorrhea may form proctitis with a thick discharge. HPV (human papillomavirus)may form perianal warts. Enterobius vermicularis, also known as pinworm or seatworm, is perhaps the most prevalent parasite in the United States. It is estimated that 20% to 30% of all children are infected with pinworms, and a total of 30 to 40 million persons are infected.

137 367. In differentiating a patient with cholangitis from one with simple cholecystitis, the most helpful finding is: a. fever. b. hyperlipasemia. c. jaundice. d. Murphy’s sign. e. elevated serum aminotransferases. 368. A 27 year-old man complains of fever and right upper quadrant pain. He is not jaundiced, and his urine shows only trace bilirubin when dipped. Both ALT and AST are 50 to 60 times the norm. You suspect: a. hemolysis. b. pancreatic disease. c. biliary disease. d. liver disease. e. stomach cancer. 369. The most sensitive and specific test for diagnosing acute cholecystitis is: a. Nuclear scintigraphy with technetium-99m-labeled IDA (iminodiacetic acid) b. ultrasound. c. oral cholecystogram. d. non-contrast CAT scan of abdomen. e. oral and intravenous contrast CAT scan of abdomen. 370. The test of choice for detecting the presence of gallstones is: a. IDA scan. b. non-contrast CAT scan of abdomen. c. oral and intravenous contrast CAT scan of abdomen. d. oral cholecystogram.(OCG) e. ultrasound.(US) 371. Lipase: a. hydrolyzes triglycerides. b. exists only in the pancreas. c. in acute pancreatitis peaks in 3 hours and levels fall over 24-48 hours. d. degree of elevation can be used as a marker of pancreatitis disease severity. e. is definitely an inferior test to amylase levels for pancreatitis. 372. This process in a 2-year-old child may be associated with: a. celiac sprue. b. Crohn’s disease. c. cystic fibrosis. d. Hirschsprung’s disease. e. intussusception. 373. The predominant symptom in a patient with diverticulitis is: a. abdominal pain. b. bloody stools. c. constipation. d. diarrhea. e. tenesmus.

Rosen 5th, Figure 91-13

138

367. c th

Rosen 5 , Chapter 85, p.1267, 1269

368. d Rosen 5th, Chapter 85, p. 1253

369. a Rosen 5th, Chapter 85, p. 1267

370. e th

Rosen 5 , Chapter 85, p. 1266

371. a Rosen 5th, Chapter 86, p. 1276

372. c Rosen 5th, Chapter 91, p. 1356

373. a Rosen 5th, Chapter 90, p. 1330

Although patients with cholangitis , in general, will have a higher fever and appear more ill than those with cholecystitis, there can be considerable variability and overlap. The presence of jaundice is the clinical sign most helpful in differentiating these two disorders. An elevated bilirubin is characteristic of cholangitis and uncommon in cholecystitis. Elevated serum aminotransferases may be found in both conditions. The typical patient with hepatitis has elevations (tenfold to 100-fold) of serum aspartate aminotransferase (AST) and alanine aminotransferase (ALT), with ALT generally elevated in excess of AST. Bilirubin may be moderately increased, but typically emerges several days to a week or more after the onset of clinical symptoms. Alkaline phosphatase and lactate dehydrogenase may be elevated but are rarely more than two to three times normal. Nuclear scintigraphy using technetium-99m-labeled iminodiacetic acid (IDA) is generally considered the most sensitive and specific test for cholecystitis. Ultrasound imaging is the most useful test in the ED setting. Ultrasound is the procedure of choice for investigating the gallbladder. US imaging can be performed rapidly, does not require the overnight delay necessary with an oral cholecystogram, is at least as sensitive, and provides the added use of permitting evaluation of surrounding structures. Oral cholecystogram remains an option for the patient with clinical suspicion of cholelithiasis in the uncommon circumstance when US imaging has failed to identify the gallbladder. Lipase is a pancreatic enzyme that that hydrolyzes triglycerides. Lipase, like amylase, exists in other tissue. In the presence of pancreatic inflammation it peaks at 24 hours and the levels fall over 8 to 14 days. The degree of elevation of amylase or lipase is not a marker of disease severity. Several expert authors recommend using lipase over amylase when seeking the diagnosis of pancreatitis. Rectal prolapse, or procidentia, is a disease of the extremes of age. In children, procidentia may herald the presence of malnutrition or cystic fibrosis, and occurs during the first 2 years of life. Boys are more commonly affected than girls. Children usually have a mucosal prolapse. The parent reports protrusion during defecation with small amounts of mucus or blood. In adults complete procidentia is most common in older women with a history of excessive straining. The cause is a laxity of attachment structures and it is often accompanied by uterine prolapse or a cystocele. The predominant symptom in patients with classical sigmoid diverticulitis is persistent abdominal pain . Initially the pain may be vague and generalized, but it quickly becomes well localized to the left lower quadrant. Low grade fever, malaise, and a change in bowel habits, usually constipation, are common.

139 374. A 48 year-old man has severe abdominal pain that began suddenly as he was driving to work this morning. He also complains of vomiting, which you see is not bilious. He has diffuse tenderness on exam. His WBC count is 14,300 / mm. You suspect this is a surgical process rather than medical because: a. he has a fever. b. his pain is diffuse. c. his white blood cell count is elevated. d. the pain started before the vomiting. e. the vomiting started before the pain. 375. A 22 year-old homosexual male complains of rectal itching and a purulent discharge. Anoscopy shows a proctitis. You suspect the causative organism to be: a. Campylobacter. b. Chlamydia. c. Giardia. d. N. gonorrhoeae. e. Herpes simplex. 376. Which statement is correct concerning the patient with a small bowel obstruction? a. increase in intraluminal pressure result in capillary and lymphatic obstruction. b. early bowel distention decreases epithelial secretory activity and decreases the amount of fluid in the bowel lumen. c. even with aggressive treatment, there is little change in the mortality of patients with small bowel obstruction. d. peristalsis decreases during the early stages of the disease process. e. all complete small bowel obstructions require emergent surgery. 377. The most common clinical manifestation of cholelithiasis is: a. gallbladder carcinoma. b. biliary colic. c. cholecystitis. d. cholangitis. e. pancreatitis. 378. Traveler’s diarrhea is most commonly caused by : a. Shigella. b. Campylobacter. c. Giardia lamblia. d. rotavirus. e. enterotoxigenic E. coli. 379. Acute mesenteric ischemia is most commonly caused by: a. arterial thrombosis. b. arterial embolus. c. venous occlusion. d. hypercoagulable state. e. nonocclusive vascular disease.

140

374. d th

Rosen 5 , Chapter 22, p. 192

375. d Rosen 5th, Chapter 91, p. 1353

Surgical causes of abdominal pain are more likely to present with pain first, followed by nausea and vomiting, rather than the converse. Diffuse pain is generally nonsurgical, but may represent the early visceral component of a surgical process. Fever does not accurately predict significant surgical pathology. Serial white blood cell counts have failed to discriminate surgical versus nonsurgical conditions. Gonorrhea causes proctitis that results from anal intercourse or autoinoculation from vaginal secretions after a 5- to 7-day incubation period. Proctitis may cause bloody or purulent rectal discharge , tenesmus and pruritus ani.

As intraluminal pressure increases it causes capillary and lymphatic obstruction with subsequent bowel wall edema. Early bowel distension stimulates epithelial cell Rosen 5 , secretory activity. Mechanical small bowel obstruction stimulates peristalsis. Chapter 87, Aggressive treatment has decreased mortality from approximately 60% in 1900 to less p. 1283, 1284, & 1287 than 5 % today. Up to 35 % to 50% of patients with complete obstruction will have resolution of symptoms with non-surgical intervention. 377. b Although cholelithiasis is associated with cholecystitis, cholangitis, gallbladder carcinoma, and pancreatitis, its most common clinical manifestation is biliary colic. Rosen 5th, Chapter 85, p. 1265 378. e Approximately 80 % to 85% of traveler’s diarrhea is caused by bacteria. An estimated 45%-50% is caused by enterotoxigenic E. coli, 8%-12% by Shigella and 7%-9% by Rosen 5th, Campylobacter. Rotavirus accounts for 5%-10% and Giardia lamblia accounts for Chapter 89, p. 1321,1323 4%-5%. 379. b Although acute mesenteric ischemia may be caused by mesenteric arterial thrombosis, nonocclusive mesenteric ischemia, mesenteric venous thrombosis and hypercoagulable Rosen 5th, states (causing a venous thrombosis), the most common cause is an arterial embolus. Chapter 87, p. 1288-1289

376. a th

141 380. The most predictive finding for appendicitis is: a. pain for more than 48 hours. b. migration of initial periumbilical pain to the right lower quadrant. c. diffuse abdominal pain. d. lack of abdominal rigidity. e. history of similar episodes. 381. Cecal volvulus is most common in: a. infants. b. young children. c. institutionalized teenagers. d. age 25 to 35 years. e. the elderly. 382. Regarding nasogastric(NG) tubes and lavage: a. NG tubes do not aggravate hemorrhage from varices or Mallory-Weiss tears. b. Iced water lavage is recommended in patients with upper GI bleeding. c. Sterile fluid must be used as an irrigant. d. Gastric lavage has been shown to reduce blood loss in patients with upper GI bleeding. e. Gastric lavage has been shown to be completely safe with no serious complications reported. 383. Vomiting secondarily to Bacillus cereus is almost always from eating contaminated: a. raw oysters. b. poultry. c. dairy products. d. raw fish. e. fried rice. 384. Meperidine (Demerol®) : a. has a longer duration of action than morphine. b. has a metabolite that may cause CNS toxicity. c. has no significant interaction with MAO inhibitors. d. has been conclusively shown to be superior to morphine for treating biliary colic. e. can be used repeatedly in the elderly without undue concern. 385. When considering abdominal pain: a. somatic pain is dull, constant and poorly localized. b. somatic pain is intense, intermittent and poorly localized. c. somatic pain is intense, constant and well localized. d. visceral pain is dull, intermittent and well localized. e. visceral pain is intense, constant and well localized. 386. In children, gastroenteritis accompanied by seizures should suggest infection with: a. Campylobacter. b. E. coli 0157:H7. c. Giardia. d. Salmonella. e. Shigella.

142

380. b th

Rosen 5 , Chapter 88, p. 1294

381. d th

Rosen 5 , Chapter 90, p. 1335

382. a th

Rosen 5 , Chapter 23, p. 197

383. e Rosen 5th, Chapter 89, p. 1309

384. b th

Rosen 5 , Chapter 182, p. 2562-2563

385. c Rosen 5th, Chapter 22, p. 190

386. e Rosen 5th, Chapter 89, p. 1304-1305

Three findings that are highly predictive of appendicitis: right lower quadrant pain, rigidity, and migration of initial periumbilical pain to the right lower quadrant.

Volvulus of the cecum occurs in all ages but is most common in persons 25 to 35 years of age. Unlike in sigmoid volvulus, severe, chronic constipation is not an underlying factor, and there is no association with psychiatric or neurological diseases. No evidence exists that gastric tube placement aggravates hemorrhage from varices or Mallory-Weis tears. Gastric lavage does not reduce blood loss in patients with UGIB, and iced lavage is not recommended. The irrigant need not be sterile. Gastric tubes are safe in most patients but pharyngeal and esophageal perforation, cardiac arrest, ethmoid sinus fracture with brain trauma and bronchial intubation have all been reported. The emetic form of Bacillus cerus illness is almost always caused by the ingestion of contaminated fried rice.

Meperidine’s duration of action is no longer than morphine’s and most studies have not shown it to be of a marked advantage for use in biliary tract disease or pancreatitis. It may have a fatal interaction with MAO inhibitors. It has a CNS toxic metabolite that is renally excreted and that also has a longer half-life in the elderly. Visceral pain results from stimulating autonomic nerves invested in the visceral peritoneum surrounding internal organs. It is often the earliest manifestation of a particular disease process This discomfort is poorly characterized and difficult to localize. If the involved organ is affected by peristalsis, the pain is often described as intermittent, crampy, or colicky. Somatic pain occurs with irritation of the parietal peritoneum. Somatic pain is often described as intense ,constant and better localized In children it is common for shigellosis to cause convulsions.

143 387. You have just seen 4 adolescents from the same 7th grade class school with apparent appendicitis – right lower abdominal pain, anorexia, nausea, and low-grade fever. You suspect they may actually be infected with: a. Bacillus cereus. b. E. coli 0157:H7. c. Isosporidiosis. d. epidemic Bunyavirus. e. Yersinia enterocolitica. 388. A preferred oral therapy for antibiotic-induced pseudomembranous enterocolitis is: a. a 2nd or 3rd generation fluoroquinolone. b. ampicillin. c. chloramphenicol. d. metronidazole. e. Pepto-Bismol®. 389. Lactulose is useful in patients with hepatic encephalopathy because it: a. acidifies the fecal stream, resulting in the trapping of ammonia as ammonium in the stool. b. causes ammonia to be transmitted to the intracellular region through the sodium-potassium membrane pump; once there, it is safely used in the Kreb’s cycle. c. forces renal excretion of sodium bicarbonate, leading to compensatory hyperpnea and respiratory alkalosis, which keeps ammonia nontoxic. d. provides an alternative calorie source to ammonia, preventing its digestion and the subsequent metabolic ketoacidosis. e. works in a manner as yet undescribed. 390. A 47-year-old man complains of right upper quadrant pain, vomiting after meals, and dark urine. Laboratory studies show AST 420, ALT 198. This is highly suggestive for: a. obstructive biliary disease. b. pancreatitis. c. hepatitis B. d. hepatitis C. e. alcohol related liver disease. 391. From an Emergency Practitioner’s perspective, the most acutely devastating complication of ulcerative colitis is: a. carcinoma. b. fistula formation. c. hepatorenal syndrome. d. malabsorption and electrolyte abnormalities. e. toxic megacolon. 392. A 72 year-old bedridden patient is sent for evaluation of vomiting. On physical exam you find several well-healed scars, a distended tympanic abdomen, quiet bowel sounds, and an empty rectal vault. The plain film radiograph of the abdomen shows a markedly dilated single loop of colon on the left side suggestive of a sigmoid volvulus. You need to consult a: a. surgeon for laparotomy, detorsion, and reattachment of the sigmoid in the left lower quadrant. b. surgeon for sigmoidoscopy, decompression and detorsion. c. surgeon for hemicolectomy. d. surgeon for sigmoid colectomy. e. surgeon for total colectomy.

144

387. e th

Rosen 5 , Chapter 89, p. 1305

388. d Rosen 5th, Chapter 89, p. 1312-1313

389. a th

Rosen 5 , Chapter 85, p. 1260

390. e Rosen 5th, Chapter 85, p. 1258

391. e Rosen 5th, Chapter 90, p. 1338

392. b Rosen 5th, Chapter 87, p. 1334

The initial clinical picture of Yersinia enterocolitis resembles that of infection by other invasive intestinal organisms: fever; colicky abdominal pain; watery, greenish, and sometimes bloody diarrhea; and constitutional symptoms of anorexia, vomiting, and malaise. A substantial number of patients with yersiniosis, in particular adolescents and young adults, develop an ileocecitis. In these cases, lower abdominal pain with little or no diarrhea predominates and may perfectly mimic acute appendicitis. Large gastrointestinal outbreaks have been traced to contaminated milk, largely because physicians noticed an extraordinary jump in the number of negative appendectomies. Many cases of antibiotic-associated colitis are self-limited, provided that the offending agent is discontinued. When antibiotic treatment for C. difficle is warranted, oral metronidazole is typically used unless the patient is significantly ill and merits IV therapy. Lactulose is a poorly absorbed sugar metabolized by colonic bacteria yielding lactic acid. The salutary effects of this agent are related both to the acidification of the fecal stream, resulting in the trapping of ammonia as ammonium in the stool, and to its cathartic action. The usual dosage of lactulose is 15 to 30 ml orally three or four times daily or in a quantity sufficient to result in several loose bowel movements daily. The principal adverse effect is excessive diarrhea, with resultant fluid and electrolyte imbalance. In alcoholic liver disease serum transaminases aspartate aminotransferase (AST) and alanine aminotransferase (ALT) are moderately elevated. Values in excess of 10 times normal are unusual and a relative predominance of AST to ALT is expected. Bilirubin is commonly elevated with a wide range of possible values. Toxic megacolon occurs in up to 5% of cases of ulcerative colitis , usually during the initial acute episode. The patient will appear septic, apathetic, and lethargic, with high fever, chills, tachycardia, and progressive abdominal pain, tenderness, and distention. The cause of toxic dilation is unknown, but precipitating factors may include use of antidiarrheal agents, vigorous use of cathartics or enemas, or barium enema examinations. Toxic dilation occurs predominantly in the transverse colon, probably because in the supine position air collects in the transverse colon. Once the diagnosis of sigmoid volvulus is made, a surgeon should be consulted. The therapy of choice for a nonstrangulationg sigmoid volvulus is decompression and detorsion, using a rectal tube via the sigmoidoscope. Success is achieved in 85% to 95% of patients.

145 393. The two most common causes of diarrhea in AIDS patients are: a. Cryptospiridium and Cytomegalovirus (CMV). b. Giardia and Rhodococcus equi. c. Salmonella and Campylobacter. d. Cyclospora and Strongyloides stercoralis. e. Toxoplasma gondii and Shigella. 394. A 70-year-old female presents with increasing constipation for the last few months. You complete her history and physical exam and become worried that she may have a serious disorder. You are most concerned about her: a. age greater than 60. b. rectal bleeding. c. abdominal distention. d. flatulence. e. depression. 395. A 28 year-old woman received a liver transplant 10 days ago. Tonight she complains of low-grade fever and body aches. Which of the following is true concerning liver post-transplant rejection? a. Rejection often begins 1 to 2 weeks post-op. b. Leukocytosis may occur. c. Fever may occur. d. Right upper quadrant pain may occur. e. All of the above are true 396. The most common cause of acute food poisoning in the United States is: a. Bacillus cereus. b. Clostridium perfringens. c. Escherichia coli 0157:H7. d. Staphylococcal organisms. e. Yersinia. 397. A 51 year-old businessman ate sushi several hours before he started vomiting and having crampy, explosive diarrhea. He requires intravenous rehydration and anti-emetics. He now feels better and asks for ice chips. You notice he spits them out, explaining, “They burn my mouth, and my teeth feel loose.” He is probably suffering from: a. ciguatera poisoning. b. scombroid poisoning. c. tetrodotoxin psychosis. d. E. coli 0157:H7 poisoning. e. disulfiram (Antabuse®) reaction. 398. You can tell the suffering gentleman with the condition described in question #397 that he: a. will probably require psychiatric counseling. b. must wait for the antidote to be delivered from Japan. c. can expect rapid improvement. d. must avoid alcohol, as it can worsen his symptoms. e. will need high-dose steroid therapy.

146

393. a th

Rosen 5 , Chapter 89, p. 1319

394. b th

Rosen 5 , Chapter 2, p. 210

395. e th

Rosen 5 , Chapter 178, p. 2508

396. b th

Rosen 5 , Chapter 89, p. 1308

397. a th

Rosen 5 , Chapter 89, p. 1311

398. d Rosen 5th, Chapter 89, p. 1311

Cryptosporidium and cytomegalovirus (CMV) infections are the most causes of diarrhea in AIDS patients. The incidence of each is 15% to 40%.Chronic persistent diarrhea is most often from one coccidia, Cryptosporidium or Isospora belli The most concerning symptoms associated with constipation are rectal bleeding and change in the caliber of the stool. These are warning signals suggesting possible colorectal cancer. Rejection often occurs 1 to 2 weeks after surgery, with fever, right upper quadrant pain, and elevated bilirubin and transaminases. Leukocytosis may occur but is nonspecific. Clostridium perfringens is probably the most common cause of acute food poisoning in the US, constituting almost one fourth of all bacteria-associated food-borne illnesses and usually occurring in large outbreaks caused by ingestion of meat or poultry dishes. Classically, patients with ciguatoxin develop both GI and neurological symptoms. The GI symptoms (e.g., nausea, vomiting, profuse watery diarrhea, crampy abdominal pain, and diaphoresis) tend to appear earlier. The panoply of neurological symptoms consists largely of dysesthesias and paresthesias around the throat and the perioral area; “burning feet,” which may resemble alcoholic peripheral neuropathy; “loose, painful teeth”; and sometimes CNS changes, such as ataxia, weakness, vertigo, visual hallucinations, and even confusion and coma. One symptom highly suggestive of ciguatera fish poisoning is sensory reversal dysesthesia, in which cold objects are perceived to be warm and vice versa. Patients describe such distortion of temperature perception very vividly. Another classic feature is either a return or a worsening of the symptoms after ingestion of alcohol. Ciguatera poisoning lasts an average of 1 to 2 weeks, but at least half of its victims are still symptomatic at 8 weeks. The neurological symptoms, particularly the paresthesias and dysesthesias, tend to persist longer than the GI symptoms and have been reported up to years later.

147 399. When assessing a patient for acute appendicitis, which of the following describes the obturator sign? a. pain produced by internal rotation of the flexed right hip. b. pain produced by external rotation of the flexed right hip. c. pain produced by extension of the right hip with the patient in the left lateral decubitus position. d. sensation of pain in the right lower quadrant with palpation of the left lower quadrant. e. pain produced by the patient coughing. 400. A type of hemorrhoid that requires emergent surgical consultation is: a. thrombosed external hemorrhoid. b. second –degree internal hemorrhoid. c. third-degree internal hemorrhoid. d. nonthrombosed fourth degree internal hemorrhoid e. thrombosed or gangrenous fourth degree internal hemorrhoid.

148

399. a th

Rosen 5 , Chapter 88, p. 1294

400. e Rosen 5th, Chapter 91, p. 1347

The obturator sign is the elicitation of pain as the right hip is flexed and internally rotated. Other clinical signs of acute appendicitis include the psoas sign (increase of pain when the psoas muscle is stretched as the patient is asked to extend his/her right hip), and Rovsing's sign ( referred right lower quadrant pain with palpation on the left lower quadrant). Patients with acute, gangrenous, fourth-degree internal hemorrhoids should be referred for emergent hemorrhoidectomy. Nonthrombosed fourth-degree hemorrhoids require nonemergent hemorrhoidectomy. Acutely thrombosed external hemorrhoids can be excised in the ED for pain relief. Second- and third- degree internal hemorrhoids require referral for elective surgery.

149

4.0 – Cutaneous Disorders 502. A 25-year-old woman is worried about some patches on her back that did not tan this summer. She probably has: a. pityriasis rosea. b. seborrheic dermatitis. c. tinea versicolor. d. vitiligo. e. xerosis. 503. A 27-year-old woman has several dozen small, scaly, slightly itchy lesions on her back. She also points out a 3-cm lesion that she noticed a few days before the above rash. You can tell her that: a. she may get some lesions in her mouth. b. the rash is likely to reoccur every year or two. c. chronic low-dose griseofulvin will be curative. d. its cause is unknown. e. topical hydrocortisone will shorten the course of the disease. 504. A wheal is: a. an elevated skin lesion >0.5 cm in diameter and depth. b. a nodule filled with expressible material. c. a blister >0.5 cm in diameter filled with clear fluid. d. a blister filled with cloudy or purulent fluid. e. a papule or plaque of dermal edema; often with central pallor and irregular borders. 505. A 54-year-old woman recently started trimethoprim-sulfamethoxazole for a sinus infection. She complains of low-grade fever, lethargy, loss of appetite, and this rash (Figure 114-13). She has similar lesions in her mouth. a. She requires intravenous broad-spectrum antibiotic coverage to prevent progression to toxic shock syndrome. b. This is Kaposi’s sarcoma; she probably has AIDS. c. The severe form of this disease is called a StevensJohnson syndrome. d. Systemic steroids will shorten the course of the disease. e. It is caused almost exclusively by viral infections Rosen 5th, Figure 114-13 506. A 3-year-old boy has had several days of yellow crusting lesions on his forehead and chin. They are itchy but not painful. The child is afebrile and nontoxic. a. Group A streptococcus is the most common causative organism. b. Systemic antibiotics have a higher cure rate than topical agents. c. These lesions are not contagious. d. Systemic antibiotics will prevent later occurrence of glomerulonephritis. e. Without treatment, the lesions heal within 3 to 6 weeks.

150

502. c th

Rosen 5 , Chapter 114, p. 1637

503. d Rosen 5th, Chapter 114, p. 1638

504. e Rosen 5th, Chapter 114, p. 1636

505. c Rosen 5th, Chapter 114, p. 1648

506. e Rosen 5th, Chapter 114, p. 1639

Tinea versicolor is a superficial yeast infection caused by Pityrosporum ovale. Superficial scaling patches occur mainly on the chest and trunk but may extend to the head and limbs. As the name implies, lesions can be a variety of colors including pink, tan, or white. The disease may be associated with pruritus, but medical care is often sought because the spots do not tan. On physical examination, a fine subtle scale is noted that may appear hypopigmented. A KOH preparation reveals short hyphae mixed with spores (“chopped spaghetti and meatballs”). Pityriasis rosea is a mild skin eruption predominantly found in children and young adults. The lesions are multiple pink or pigmented oval papules or plaques 1 to 2 cm in diameter on the trunk and proximal extremities. Mild scaling may be present. The lesions are parallel to the ribs, forming a Christmas tree–like distribution on the trunk. Oral lesions are rare. In children, papular or vesicular variants of the disease may occur. In half the cases, the generalized eruption is preceded by a week by the appearance of a “herald patch.” This is a larger lesion, 2 to 6 cm in diameter, that resembles the smaller lesions in other respects. The eruption is usually asymptomatic, although pruritus may be present. Pityriasis rosea is self-limited, resolving in 8 to 12 weeks. Its cause is unknown, although a virus is suspected. Treatment is usually unnecessary, except for symptomatic alleviation of bothersome pruritus. Macule: Flat, color differs from surrounding skin Patch: A macule with surface changes (i.e., scale or wrinkling) Papule: Elevated skin lesion 0.5 cm in diameter; without substantial depth Nodule: Elevated skin lesion >0.5 cm in diameter and depth Cyst: Nodule filled with expressible material Vesicle: Blisters 0.5 cm in diameter filled with clear fluid Pustule: Vesicle filled with cloudy or purulent fluid Crust: Liquid debris that has dried on the skin surface Scale: Visibly thickened stratum corneum; usually white Lichenification: Epidermal thickening characterized by visible and palpable skin thickening and accentuated skin markings Induration: Dermal thickening that feels thick and firm Wheal: Papule of dermal edema; often with central pallor and irregular borders Erythema: Red appearance of skin caused by vasodilatation of dermal blood vessels Erythema multiforme is an acute, usually self-limiting disease precipitated by a variety of factors. It is characterized by the sudden appearance of skin lesions that are erythematous or violaceous macules, papules, vesicles, or bullae. Their distribution is often symmetrical, most commonly involving the soles and palms, the backs of the hands or feet, and the extensor surfaces of the extremities. Treatment should begin with a search for the underlying cause. Mild forms resolve spontaneously in 2 to 3 weeks. Systemic steroids are commonly used and provide symptomatic relief, but are of unproven benefit in influencing the duration and outcome. Impetigo is a slowly evolving pustular eruption, most common in preschool children. Currently, Staphylococcus aureus is the most common pathogen, with Group A streptococcus a distant second. Both systemic and topical therapy is equally successful in treating impetigo. Lesions are very contagious among infants and young children and less so in older children and adults. There is no evidence, however, that systemic antibiotics prevent the development of acute glomerulonephritis. Without treatment, impetigo heals within 3 to 6 weeks.

151 507. A 21-year-old man has fever and a urethral discharge. He also complains of a swollen painful left knee and this rash. a. He can be treated as an outpatient with oral antibiotics. b. This condition occurs more in men than in women. c. If the lesions are cultured, they are usually positive for the organism. d. The tick serves as a vector for this disease. e. You expect the rash to be pustular rather than petechial and hemorrhagic. 508. The most common medication-related cause of erythema nodosum is: a. sulfa-based drugs. b. non-steroidal anti-inflammatory agents. c. oral contraceptives. d. penicillin. e. fluoroquinolones. 509. The most common skin eruption due to drugs is: a. bullae. b. macules. c. papules. d. striae. e. urticaria. 510. A 25-year-old woman had an abortion five days ago. She now has fever and a flu-like syndrome. Today she started vomiting and got confused. Oral temperature 103.4°F, heart rate 120 / minute, blood pressure 80/40 mmHg. You also notice diffuse eryhtema. You know that: a. mucous membrane involvement would indicate this is not toxic shock syndrome. b. you can also see pharyngitis, a “strawberry tongue,” conjunctivitis, and vaginitis with this illness. c. supportive care and high-dose penicillin is the appropriate treatment. d. desquamation tends to spare the hands and feet. e. the skin rash may take several weeks to dissipate. 511. Nikolsky’s sign is often found in patients with: a. acanthosis nigrans. b. bullous pemphigoid. c. hidradenitis suppurativa. d. Rocky Mountain spotted fever. e. toxic epidermal necrolysis. 512. Erysipelas is usually caused by: a. Neisseria gonorrhoeae. b. Coxsackievirus. c. Bartonella tularensis. d. Group A streptococcus. e. Staphylococcus epidermidis.

152

507. e th

Rosen 5 , Chapter 114, p. 1640

508. c Rosen 5th, Chapter 114, p. 1653

509. e Rosen 5th, Chapter 114, p. 1640

510. b Rosen 5th, Chapter 114, p. 1644

511. e Rosen 5th, Chapter 114, p. 1643

512. d Rosen 5th, Chapter 131, p. 1946

About 2% of patients with mucosal gonococcal infection will develop disseminated GC infection. Hospitalization is recommended for patients in whom the diagnosis is uncertain and for those who have septic arthritis, meningitis, or endocarditis. It affects women primarily. Fever and migratory polyarthralgias commonly accompany the skin lesions. The rash of disseminated gonococcemia is pustular rather than obviously petechial and hemorrhagic, as are the lesions of Rocky Mountain spotted efevr and meningococcemia. A number of underlying conditions produce erythema nodosum: tuberculosis, sarcoidosis, coccidioidomycosis, histoplasmosis, ulcerative colitis, regional enteritis, pregnancy, infections with streptococci, Yersinia enterocolitica, and chlamydia. As with erythema multiforme, many cases of erythema nodosum are idiopathic. Oral contraceptive agents are the leading cause of drug-induced cases. The differential diagnosis includes traumatic bruises and subcutaneous fat necrosis. A given drug can produce a skin eruption of a different appearance in different patients or a different appearance in the same patient on different occasions. The most common eruptions are urticaria (hives) and morbilliform rashes. Drug reactions tend to appear within a week after the drug is taken, with the exception of reactions to semisynthetic penicillins, which commonly occur later. Skin lesions may appear after a drug has been discontinued and may worsen if the drug or its metabolites persist in the system. Special note should be made of penicillin because it is the most common cause of drug reaction. Serum sickness and urticaria are the most common manifestations of penicillin allergy. In toxic shock syndrome, the rash is typically diffuse, blanching, macular erythroderma. Accompanying nonexudative mucous membrane inflammation is common. Pharyngitis, sometimes accompanied by a “strawberry tongue,” conjunctivitis, or vaginitis may be seen. As a rule, the rash fades within 3 days of its appearance. This is followed by a full-thickness desquamation, most commonly involving the hands and feet. Initial treatment of TSS consists of IV fluid replacement, ventilatory support, pressor agents, penicillinase-resistant antibiotics, and drainage of infected sites. Corticosteroids reduce the severity of illness and duration of fever if initiated within 2 to 3 days after the onset of illness. Nikolsky’s sign is epidermal shearing following lateral pressure on unblistered skin. The extension of a blister following application of pressure to its roof has earned the eponym Asboe-Hansen’s sign. Nikolsky's sign is most commonly found in scalded skin syndrome (also called Ritter disease), toxic epidermal necrolysis, and pemphigus vulgaris. Erysipelas is an acute superficial cellulitis characterized by a sharply demarcated border surrounding skin that is raised, deeply erythematous, indurated, and painful. It usually involves the dermis, lymphatics, and most of the superficial subcutaneous tissue. Erysipelas most often occurs in the very young and those age 50 to 60 years; it is associated with small breaks in the skin, nephrotic syndrome, and postoperative wounds. Patients usually appear toxic with a prodrome of fever, chills, and malaise preceding the eruption of a bright red cellulitis predominantly on the lower extremities or on the face. Group A Streptococcus, other streptococcal species, and S. aureus are the involved pathogens.

153 513. A positive Hutchinson sign is found in patients with: a. herpes zoster ophthalmicus. b. Lyme disease. c. measles. d. mumps. e. Rocky Mountain spotted fever. 514. Physical findings of patients with scarlet fever and Kawasaki disease are similar. The finding most likely to make you think of scarlet fever would be: a. ‘strawberry’ tongue. b. cervical adenopathy. c. fever. d. generalized erythematous rash. e. circumoral pallor. 515. There is an outbreak of head lice in the local school district, and the local school nurse calls for advice. You tell her that: a. household contacts require treatment. b. involvement of the interdigital web spaces is typical. c. nits are seen more frequently than the adult louse form. d. pediculosis capitis is more common in adults. e. the organism is a mite. 516. Oral acyclovir is recommended for the treatment of an initial episode of: a. genital herpes simplex. b. herpes zoster. c. oral herpes simplex. d. varicella. e. vaccinia. 517. Roseola infantum: a. causes a high fever followed in several days by a desquamating rash. b. frequently is a precursor to encephalitis. c. generally occurs in school-age children. d. is also called fifth disease. e. is caused by a herpes virus. 518. A pinkish maculopapular rash that first appears on the face and associated with generalized lymphadenopathy including suboccipital and postauricular nodes is most typical of: a. measles b. Rocky Mountain spotted fever. c. roseola. d. rubella. e. rubeola. 519. The primary lesions of pemphigus vulgaris are: a. bullae. b. macules. c. nodules. d. papules.

154 e. wheals.

155

513. a th

Rosen 5 , Chapter 66, p. 918

514. e Rosen 5th, Chapter 123, p. 1804-1805 Chapter 114, p. 1646

515. c Rosen 5th, Chapter 114, p. 1649

516. a Rosen 5th, Chapter 124, p. 1819

517. e th

Rosen 5 , Chapter 114, p. 1822

518. d Rosen 5th, Chapter 114, Chapter 128, pages listed in text of answer

519. a Rosen 5th, Chapter 114, p. 1653-1654

Herpes zoster keratoconjunctivitis occurs as a result of activation of the virus along ophthalmic division of the trigeminal nerve. The rash follows dermatomal patterns, involves the forehead and upper eyelid, and produces significant pain. Involvement of the nasociliary nerve, manifested by zoster lesions on the tip of the nose (Hutchinson’s sign), is associated with a 76% risk of ocular involvement vs. 34% risk if the nerve is not involved. Kawasaki disease, or mucocutaneous lymph node syndrome (MLNS) is a disease of unclear etiology found predominantly in children less than 9 years of age. The diagnosis of this disorder is based on a prolonged fever associated with at least four of the following: (1) conjunctivitis, (2) rash, (3) lymphadenopathy, (4) changes in the oropharynx consisting of injection of the pharynx and lips with prominent papillae of the tongue (strawberry tongue), and (5) extremity erythema and edema. Patients with scarlet fever can have the erythematous rash, strawberry tongue, lymphadenopathy, and fever, but also have facial flushing with circumoral pallor. Only occasionally are the adult lice forms found, looking like blue or black grains. The nits often attach to the base of the hair shafts, appearing as white dots. Any sexual contacts should be treated; household contacts should be examined, and if uninfected, no treatment is necessary. Scabies involves the interdigital web spaces and is a mite infestation. Acyclovir is not routinely recommended for oral herpes, herpes zoster, or varicella infections unless the patient is immunocompromised. Acyclovir reduces the duration of viral shedding and accelerates healing. It does not prevent recurrent episodes. Roseola infantum is the most common exanthem of children younger than 2 years of age and occurs most often at about 1 year of age. The illness begins abruptly with the acute onset of fever, often as high as 41o C, lasting three to five days. Despite the fever, the child usually remains active and alert. A fine, evanescent, rose-colored maculopapular rash then appears on the trunk after lysis of the fever, which may last for 1 or 2 days. The rash may spread to the face and extremities. This is a typical picture of rubella (German measles), particularly the involvement of the postauricular nodes. It is important to recognize rubella in order to avoid maternal exposure that can result in severe birth defects (p. 1825). Roseola infantum is described in question #517 (p. 1822). Rocky Mountain spotted fever is manifested by the abrupt onset of fever, headaches, chills, and malaise; rash develops on the second to fourth day, with blanching erythematous macules around the wrists and ankles. They may become petechial (p. 1891-1896). Rubeola (measles) is a febrile illness, with rash that starts on the face and spreads to the trunk on the third to fifth day of illness. Koplik's spots-bright red spots with bluish centers opposite the molars-occur early in the course of the measles (p. 1827). Pemphigus vulgaris (PV) is an uncommon, but important, dermatologic disorder. The mortality rate before the use of steroids was approximately 95%. PV is a bullous disease, affecting both sexes equally, and is most common in patient 40 to 60 years of age. Nikolsky’s sign is present and characteristic of the disease.

156

520. Hidradenitis suppurativa: a. begins as a painful deep erythematous nodule usually in the axilla or groin. b. begins in childhood. c. is easily treated with 10-days of a long-acting tetracycline. d. is more common in men. e. should never be drained in the emergency department.

157

520. a th

Rosen 5 , Chapter 114, p. 1639

Hidradenitis suppurativa affects the apocrine sweat glands. Recurrent abscess formation in the axillae and groin resembles localized furunculosis. The condition tends to be recurrent and may be extremely resistant to therapy. It is treated with drainage of abscesses, but antistaphylococcal antibiotics are useful if administered early and for a prolonged period.

158

5.0 – Endocrine, Metabolic, Nutritional Disorders 521. The most common cause of coma in a patient with diabetes is: a. alcohol intoxication. b. diabetic ketoacidosis. c. hypoglycemia. d. sepsis. e. stroke. 522. In a hypoglycemic patient who has known adrenal insufficiency , you should consider giving intravenous: a. ACTH. b. amiodarone. c. hydrocortisone. d. sodium bicarbonate. e. thyroxin. 523. The only agents capable of stimulating hepatic glucose production (glycogenolysis) within minutes are: a. glucagon and epinephrine. b. glucagon and insulin. c. glucagon and norepinephrine. d. insulin and epinephrine. e. insulin and norepinephrine. 524. In patients with hypoglycemia, signs of CNS dysfunction are common. The next most common physical sign would be: a. seizures. b. focal weakness. c. hypotension d. tachypnea. e. sweating. 525. The only anabolic hormone is: a. adrenaline. b. cortisol. c. glucagon. d. insulin. e. testosterone. 526. In a patient with diabetic ketoacidosis, the primary reason for mental status change is the: a. elevated blood sugar. b. metabolic acidosis. c. respiratory alkalosis. d. hypoxemia. e. elevated osmolarity.

159

521. c th

Rosen 5 , Chapter 120, p. 1748

522. c th

Rosen 5 , Chapter 120, p. 1781

523. a th

Rosen 5 , Chapter 120, p. 1745

524. e Rosen 5th, Chapter 120, p. 1748

525. d th

Rosen 5 , Chapter 120, p. 1745

526. e Rosen 5th, Chapter 120, p. 1750

The most common cause of coma associated with diabetes is hypoglycemia, due to an excess of administered insulin with respect to glucose intake. Hypoglycemia may be associated with significant morbidity and mortality. Severe hypoglycemia is usually associated with a blood sugar below 40 to 50 mg/dl and impaired cognitive function. Steroid administration should be considered for hypoglycemia that is either resistant to aggressive glucose replacement therapy or associated with the signs of adrenal insufficiency. The dose is 100 to 200 mg hydrocortisone IV in adults. The first defense against the development of hypoglycemia is a decrease in insulin secretion, but both glucagon and epinephrine are also important for the acute protection against hypoglycemia. Both of these counterregulatory hormones are the only agents capable of stimulating hepatic glucose production within minutes of their release into circulation, primarily via glycogenolysis—the release of glucose from its intracellular storage-depot glycogen. A depressed sensorium is most common. Described less frequently are seizure activity and focal neurologic findings. Sweating is common.

Insulin is the only anabolic hormone and is responsible for the metabolism and storage of carbohydrates, fat, and protein. Counterregulatory hormones include glucagon, catecholamines, cortisol, and growth hormone. Mental confusion and coma are much more likely with serum osmolarity 340 mOsm/L or higher. If the serum osmolarity is 350 mg/dL, bicarbonate >20 mEq/L, arterial pH 250 mg/dL, bicarbonate >15 mEq/L, arterial pH >7.3, moderate ketonemia c. glucose >250 mg/dL, bicarbonate 7.3, moderate ketonuria 529. The single most important initial treatment for the patient in diabetic ketoacidosis is: a. oxygen therapy. b. rapid bicarbonate administration. c. rapid fluid administration. d. rapid insulin administration. e. rapid potassium replacement. 530. In a patient with diabetic ketoacidosis, the preferred method of insulin administration is: a. intramuscular. b. intravenous bolus – high dose. c. intravenous bolus – low dose. d. intravenous constant drip– weight-based. e. subcutaneous. 531. During treatment of diabetic ketoacidosis, the most potentially life-threatening electrolyte abnormality is: a. hypercalcemia. b. hyperkalemia. c. hypokalemia. d. hypernatremia. e. hyponatremia. 532. Concerning the use of sodium bicarbonate in the treatment of diabetic ketoacidosis, it has been shown to: a. rapidly improve the hypokalemia b. improve the central nervous system acidosis. c. improve intracellular acidosis. d. assist oxyhemoglobin dissociation. e. do none of the above – don’t use it. 533. Common presenting symptoms in a patient with alcoholic ketoacidosis include: a. blurred vision. b. delirium tremens. c. dehydration d. e.

bloody diarrhea. fetid breath odor.

161

527. a th

Rosen 5 , Chapter 120, p. 1748

528. d Rosen 5th, Chapter 120, p. 1752

529. c th

Rosen 5 , Chapter 120, p. 1752

530. d Rosen 5th, Chapter 120, p. 1752-1753

531. c Rosen 5th, Chapter 120, p. 1754

532. e th

Rosen 5 , Chapter 120, p. 1754

533. b Rosen 5th, Chapter 179, p. 2524

Urine ketone dipsticks use the nitroprusside reaction, which is a good test for acetoacetate but does not measure beta-hydroxybutyrate. Although the usual acetoacetate / beta-hydroxybutyrate ratio in diabetic ketoacidosis is 1:2.8, it may be as high as 1:30, in which case the urine dipstick does not reflect the true level of ketosis. When ketones are in the form of beta-hydroxybutyrate, the urine ketone dipsticks may infrequently yield negative reactions in patients with significant ketosis. Although the exact definition of DKA is variable, most experts agree that a blood glucose greater than 350 mg/dL, bicarbonate level less than 15 mEq/L, and an arterial pH of less than 7.3 with moderate ketonemia constitute the disease. Rapid fluid administration is the single most important initial step in the treatment of DKA. Fluid helps restore intravascular volume and normal tonicity, perfuse vital organs, improve glomerular filtration rate, and lower serum glucose and ketones. The average adult patient has a water deficit of 100 mL/kg (5 to 10 L) and a sodium deficit of 7 to 10 mEq/kg. NS is the most frequently recommended fluid for initial rehydration even though the extracellular fluid of the patient is initially hypertonic. High dosages of insulin have potentially harmful effects, including a greater incidence of iatrogenic hypoglycemia and hypokalemia. Because the half-life of regular insulin is 3 to 10 minutes, IV insulin should be administered by constant infusion rather than by repeated bolus. The current therapy of choice is regular insulin infused at 0.1 U/kg/hr up to 5 to 10 U/hr, mixed with the IV fluids. Regular insulin, 10 to 20 U/hr, administered IM accomplishes similar effect but subjects the patient to repeated painful injections. In theory, IM insulin may accumulate at a poorly perfused administration site, failing to enter the systemic circulation in a timely manner. The development of severe hypokalemia is potentially the most life-threatening electrolyte derangement during the treatment of DKA. This complication is avoidable if the pathophysiology is understood and the effects of therapy frequently monitored. To date, not a single study clearly demonstrates improved clinical outcome using bicarbonate in the treatment of DKA. Acidotic patients routinely recover from DKA without alkali therapy. Routine use of supplemental bicarbonate in the treatment of DKA is not recommended. Presenting symptoms of alcoholic ketoacidosis Nausea 76% Vomiting 73% Abdominal pain 62% Shortness of breath 20%

162 534. Initial treatment of the patient with alcoholic ketoacidosis should begin with: a. intravenous bicarbonate. b. intravenous D5NS. c. intravenous insulin. d. intravenous Ringer’s lactate. e. subcutaneous insulin. 535. In comparing patients with diabetic ketoacidosis (DKA) to those with hyperglycemic hyperosmolar nonketotic coma (HHNC), those with DKA have: a. greater fluid and electrolyte deficits. b. higher serum glucose levels. c. lower anion gaps. d. more profound acidosis. e. slower onset. 536. Na+ 136 mEq/L; K+ 4.1 mEq/L; Cl- 108 mEq/L; NaHCO3- 12 mEq/L; glucose 600 mg/dL; arterial pH 7.10 a. The anion gap is 26. b. The corrected potassium is 2.3 mEq/L. c. The corrected potassium is 5.9 mEq/L. d. The corrected sodium is 126 mEq/L. e. The corrected sodium is 152 mEq/L. 537. In rehydrating the patient with hyperosmolar hyperglycemic coma, you should use: a.D5NS. b. hypertonic saline. c.hypotonic saline. d. isotonic saline. e.Ringer’s lactate. 538. When fixing a patient’s fluid and electrolyte imbalances, they should be corrected in this order: a. sodium and chloride > volume > pH > potassium, calcium, and magnesium b. volume > pH > potassium, calcium, and magnesium > sodium and chloride c. potassium, calcium, and magnesium > volume > pH > sodium and chloride d. pH > potassium, calcium, and magnesium > sodium and chloride > volume e. volume > potassium, calcium, and magnesium > sodium and chloride > pH 539. The most common cause of metabolic acidosis in children is: a. cystic fibrosis. b. diabetes. c. febrile seizure. d. nephrotic syndrome. e. prolonged diarrhea. 540. Common precipitating factors in the development of hyperglycemic hyperosmolar nonketotic syndrome include: a.fever. b. prolonged bedrest. c.hypokalemia d. myocardial ischemia.

163 e.use of a salt substitute.

164

534. b th

Rosen 5 , Chapter 179, p. 2524

535. d Rosen 5th, Chapter 120, p. 1752

536. b Rosen 5th, Chapter 120, p. 1752

537. d Rosen 5th, Chapter 120, p. 1756

538.b Rosen 5th, Chapter 119, p. 1753-1754

539. e th

Rosen 5 , Chapter 118, p. 1718

540. c Rosen 5th, Chapter 120, p. 1756

Treatment of alcoholic ketosis is volume replacement with normal saline, glucose, thiamine, and correction of hypokalemia. This can be accomplished with 5% dextrose in normal saline and either 30 mEq of potassium chloride or 30 mEq of oral potassium. Bicarbonate is seldom necessary for the uncomplicated case but may be considered in the rare patient who has a pH of less than 7.1. If no serious complicating illness is present, the ketosis will reverse in 12 to 24 hours with this treatment. The prodrome for HHNC is significantly longer than that of DKA. The patient typically manifests more profound electrolyte imbalance and dehydration than those with DKA. A patient with HHNC does not have a ketoacidosis caused by diabetes but may have a lesser degree of metabolic acidosis. The mortality rate is much higher for HHNC as most patients are elderly and have underlying cardiac and renal disease. The reported serum sodium is often misleading in DKA. The true value of sodium may be approximated by adding 1.6 mEq/L to the reported value for every 100 mg/dl of glucose over the norm. Acidosis and dehydration contribute to high measured serum potassium despite total body deficits. Correction for acidosis can be made by subtracting 0.6 mEq/L from the laboratory value for every 0.1 decrease in pH below 7.4. Most authors agree that the use of isotonic saline (0.9% NaCl) is the most appropriate initial crystalloid for the replacement of intravascular volume. It is hypotonic to the patient’s serum osmolarity and will more rapidly restore plasma volume. Once hypotension, tachycardia, and urinary output improve, half- normal saline (0.45% NaCl) can be used to replace the remaining free water deficit. When fluids and electrolytes are altered, they should be corrected in the following orderly fashion: volume > pH > potassium, calcium, and magnesium > sodium and chloride. Equilibrium of fluid, electrolytes, and pH depends on adequate tissue perfusion and often corrects spontaneously with resolution of the underperfused state. Metabolic acidosis can be caused by one of three mechanisms: (1) increased production of acids, (2) decreased renal excretion of acids, or (3) loss of alkali. The etiologies of metabolic acidosis can be clinically divided into those that create an elevation in AG and those that do not. In the pediatric age group, dehydration from prolonged diarrhea is the most common cause of metabolic acidosis. Usually some precipitating event causes a patient to develop an insidious state of progressive hyperglycemia and hyperosmolarity, which goes unchecked. By far, acute infection is the most common precipitating cause of HHNS. Urinary tract infection and pneumonia are most common, though uremia, viral illness, and a host of metabolic and iatrogenic causes have been identified. Similarly, several drugs may predispose or contribute to hyperglycemia, volume depletion, or other effects leading to HHNS, especially diuretics, glucocorticoids, lithium, phenytoin, neuroleptics, beta-blockers, mannitol, didanosine, and calcium-channel blockers.

165 541. It is well known that vomiting leads to hypokalemia. The reason for this is: a.direct loss of potassium from stomach contents. b. hypovolemia from volume loss leads to increases in aldosterone secretion, causing the kidney to preserve sodium and bicarbonate in exchange for potassium, resulting in alkalosis which causes potassium to shift into cells in exchange for hydrogen ions. c.vomiting causes hyperventilation, leading to respiratory alkalosis and compensatory extracellular to intracellular potassium shifts. d. with the loss of hydrogen ions after a first episode of vomiting, the gastric parietal cells secrete potassium, which is lost in further vomiting. e.unknown. 542. An 86-year-old woman from a local nursing home presents in hyperosmolar hyperglycemic coma. She was recently hospitalized for pyelonephritis and bacteremia. When you look at her medication list, you suspect that her current condition is more than likely caused by: a. aspirin. b. levofloxacin. c. thyroid replacement. d. omeprazole (Prilosec®). e. hydrochlorothiazide. 543. In treating the patient mentioned in Question #542, you know that insulin a is contraindicated. b. should be given in higher doses than when treating DKA. c. can safely and effectively be used intramuscularly. d. can precipitate vascular collapse if it is given prior to volume expansion. e. can cause a potentially lethal hyperkalemia if given too rapidly. 544. The patient in Question #542 has received 2000 cc of normal saline solution and 12 units of regular insulin. Her repeat bedside glucose is 245 mg/dL. Your third liter of fluid should be: a.0.9% saline. b. 0.45% saline. c.Ringer’s lactate. d. dextrose / 0.9% saline. e.5% dextrose / 0.45% saline. 545. The most common cause of hyperthyroidism in the United States is: a.Graves’ disease. b. multinodular goiter. c.pituitary adenoma. d. thyroid cancer. e.thyroiditis. 546. A 40-year-old woman complains of palpitations. Temperature 100.8oF, heart rate 140 / minute, respiratory rate 22 / minute, blood pressure 148/80 mmHg. Her eyes are shown in this picture. You immediately suspect: a.amphetamine toxicity. b.lithium overdose. c.myxedema coma. d.pheochromocytoma.

166 e.thyroid storm.

167

541. e th

Rosen 5 , Chapter 120, p. 1755

542. b Rosen 5th, Chapter 119, p. 1728

543. d Rosen 5th, Chapter 120, p. 1756

544. e Rosen 5th, Chapter 120, p. 1756

545. a th

Rosen 5 , Chapter 173, p. 2441

546. e Rosen 5th, Chapter 122, p. 1771

Several drugs may predispose or contribute to hyperglycemia, volume depletion, or other effects leading to HHNS, especially diuretics, glucocorticoids, lithium, phenytoin, neuroleptics, beta-blockers, mannitol, didanosine, and calcium-channel blockers. The hypokalemia associated with vomiting has very little to do with the actual K+ lost in the vomitus and much more to do with the metabolic alkalosis that follows. The hypovolemia from volume loss leads to increases in aldosterone secretion, which acts on the kidney to preserve Na+ and bicarbonate in exchange for K+. The resultant alkalosis also causes K+ to shift into cells in exchange for H+. Initial volume replacement should always precede the initiation of an insulin drip in HHNS. If insulin is utilized prior to rehydration, intravascular volume may be further depleted. Insulin will cause a shift of osmotically active glucose into the intracellular space, bringing free water with it. This will further deplete the intravascular compartment and may precipitate vascular collapse. Another relative contraindication to the early use of insulin in HHNS is hypokalemia. Insulin promotes transport of potassium into cells and it should be withheld until the potassium level is known and hypokalemia has been treated. Once serum glucose decreases to less than 300 mg/dL, the intravenous solution should be changed to D5W 0.45 percent NaCl and the insulin infusion should be reduced to half or 0.05 (units/kg)/h. Graves’ disease is by far the most common cause, accounting for more than 80% of cases of hyperthyroidism in the United States. Toxic multinodular and toxic (adenoma) nodular goiters are the next most frequent causes. Less common causes of hyperthyroidism are thyroiditis, pituitary tumors, metastatic thyroid cancer, and dermoid tumors or teratomas of the ovary. Common symptoms of thyroid storm include palpitations, dyspnea, and chest pain. Fever is often present, but the tachycardia may be elevated out of proportion to the height of fever. The manifestations of ophthalmopathy include upper-lid retraction, staring, lid lag (Graefe’s sign), exophthalmos, and extraocular muscle palsies.

168 547. The most important treatment for the reduction of morbidity and mortality in thyroid storm is: a.inhibit hormone synthesis. b.block hormone release. c.prevent peripheral conversion of T4 to T3 d.block the peripheral effects of thyroid hormone. e.provide general support. 548. A drug to avoid in a patient in thyroid storm is: a. acetaminophen. b. aspirin. c. calcium chloride. d. calcium gluconate. e. insulin. 549. You are treating a patient in thyroid storm. You have given propranolol two mg intravenously. An appropriate follow-up regimen would be: a.intravenous methimazole followed by rapid intravenous sodium iodide. b. intravenous propylthiouracil (PTU) followed by slow intravenous sodium iodide. c.oral propylthiouracil followed by slow intravenous sodium iodide. d. oral sodium iodide followed by intravenous methimazole. e.oral sodium iodide followed by intravenous propylthiouracil. 550. Secondary hypothyroidism is usually caused by panhypopituitarism and initially leads to: a. thyroglobulin deficiency. b. thyroid releasing hormone deficiency. c. thyrotropin (thyroid stimulating hormone) deficiency. d. thyroxin deficiency. e. triiodothyrinine deficiency. 551. A 70-year-old patient is sent by her family doctor because “She’s just not acting right.” Rectal temperature 95.2oF, heart rate 48 / minute, blood pressure 102/72 mmHg. She is slow to respond. You notice periorbital edema and a thick tongue. Her EKG shows sinus bradycardia and low voltage, but no ischemia. Her general appearance is as shown here. In addition to abnormal thyroid function testing, you would also expect to find: a. hyperchloremia. b. hyperglycemia. c. hypocarbia. d. hypomagnesemia. e. hyponatremia. 552. Concerning the patient in Question #551, you will need to begin therapy using intravenous: a. high-dose steroids and a diuretic. b. levothyroxine and hydrocortisone. c. thyroglobulin and ACTH. d. thyrotropin and insulin. e. triiodothyrinine and a beta-blocker.

169

547. d th

Rosen 5 , Chapter 122, p. 1774

548. b Rosen 5th, Chapter 122, p. 1774

549. c Rosen 5th, Chapter 122, p. 1774

550. c Rosen 5th, Chapter 122, p. 1778

551. e Rosen 5th, Chapter 122, p. 1778

552. b Rosen 5th, Chapter 122, p. 1778-1779

Initial treatment of thyroid storm consists of stabilization, airway protection, oxygenation, intravenous fluids, and monitoring. Blockade of the peripheral adrenergic hyperactivity of thyroid crisis may be the most important factor in reducing morbidity and mortality. Beta-blockade is currently the method of choice for staunching the peripheral manifestations of thyroid storm. Propranolol can reduce dysrhythmias, hyperpyrexia, tremor, palpitations, restlessness, anxiety, and perhaps myopathy. Hyperpyrexia of thyroid storm should be treated aggressively with acetaminophen. Aspirin should not be used because it displaces thyroid hormone from thyroglobulin, thus theoretically increasing the pool of metabolically active hormone. Ice packs and hypothermia blankets may also be used. Thioamides, including propylthiouracil (PTU) and methimazole, inhibit thyroidal peroxidase, thereby preventing hormone synthesis. PTU is generally preferred over methimazole because it has the additional minor effect of inhibiting peripheral conversion of T4 to T3. PTU is given in an initial dose of 600 to 1000 mg by mouth (PO) or by nasogastric (NG) tube, followed by 200 to 250 mg every 4 to 6 hours. Further organification of iodine will be blocked within 1 hour of PTU administration, but the drug should be continued for several weeks while the hyperthyroidism is brought under control. Because preformed T4 and T3 are stored in the thyroid colloid, release of hormone can occur for weeks despite synthesis inhibition. Thus prevention of colloid hormone release is the second goal of therapy. Both iodine and lithium can inhibit thyroid hormone release. Lithium is not generally used because it can be difficult to titrate the dose, and toxic effects are common. Thioamides should be given at least 1 hour before iodine therapy to prevent organification of the iodine. Lugol’s iodine solution, 30 drops per day in 3 to 4 divided doses PO or by NG tube; potassium iodide (SSKI), 5 drops every 6 hours PO or by NG tube; or sodium iodide, 1 g slow intravenous (IV) drip every 8 to 12 hours, is acceptable. Regulation of synthesis and release of thyroid hormone is under the control of the anterior pituitary gland via thyroid-stimulating hormone (TSH), or thyrotropin. Regulation of TSH in turn is by hypothalamic thyrotropin-releasing hormone (TRH) and also by means of a feedback loop to the pituitary gland by circulating thyroxine (T4) and triiodothyrinine (T3) levels. Thyroid hormone production depends on adequate iodine intake and synthesis of thyroglobulin. Laboratory evaluation of patients with suspected myxedema coma may reveal anemia; hyponatremia; hypoglycemia; elevated transaminases, creatine phosphokinase, and lactate dehydrogenase levels; hypercholesterolemia; and arterial blood- gas abnormalities (decreased PO2 and increased PCO2). The electrocardiogram may demonstrate sinus bradycardia, prolongation of the QT interval, and low voltage with flattening or inversion of T waves. Specific therapy includes intravenous levothyroxine; an initial intravenous bolus is administered, followed by a reduced daily dose until the patient can take oral medication. This has the advantage of repleting the T4 pool and allowing the hormone to enter tissues slowly. Stress dosages of corticosteroids, such as 300 mg of hydrocortisone IV followed by 100 mg IV every 6 to 8 hours, are also routinely given because myxedema may be either a manifestation of panhypopituitarism or a coexisting condition with primary adrenal failure.

170 553. The most common precipitating factor for myxedema coma is: a. barbiturates. b. depression. c. diuretics. d. infection. e. trauma. 554. A 62-year-old man presents with classic symptoms of hypothyroidism – mild hypothermia, bradycardia, hoarse voice, and bilateral carpal tunnel syndrome. You find no signs of infection. He has an extensive cardiac history, including an automatic internal defibrillator, and is on a waiting list for heart transplant. He takes many medications, but the one you suspect is probably causing his thyroid malfunction is: a. amiodarone (Cordarone®). b. bumetinide (Bumex®). c. enalapril maleate / felodipine (Lexxel®). d. lovastatin (Mevacor®). e. sotalol (Betapace®) 555. The most common cause of acute adrenal insufficiency and adrenal crisis is a. hypothalamic infarct. b. fuunctional, from exogenous glucocorticoid administration. c. non-adrenal catecholamine-secreting tumors. d. post-partum panhypopituitarism (Sheehan’s syndrome). e. Waterhouse-Friderichsen syndrome. 556. Aldosterone is the primary mineralocorticoid. It is regulated by the renin-angiotensin system and acts to: a. change the osmotic “set-point” in the hypothalamus. b. decrease potassium excretion. c. decrease water retention. d. increase calcium reabsorption. e. increase sodium reabsorption. 557. Common laboratory findings in a patient with adrenal insufficiency include: a. high sodium, high potassium, high glucose. b. high sodium, low potassium, low glucose. c. low sodium, high potassium, low glucose. d. e.

low sodium, low potassium, high glucose. low sodium, low potassium, low glucose.

558. A 28-year-old woman has just returned from a 10-day cruise. She forgot to bring her prednisone, which she uses to treat her rheumatoid arthritis. She started vomiting yesterday and now appears quite dehydrated. Pulse 120 / minute, blood pressure 84/42 mmHg. The best way to treat her hypotension is: a. corticosteroids. b. intravenous fluids and corticosteroids. c. intravenous fluids and pressor agents. d. intravenous fluids. e. pressor agents.

171

553. d th

Rosen 5 , Chapter 122, p. 1778

554. a Rosen 5th, Chapter 122, p. 1775

555. b th

Rosen 5 , Chapter 122, p. 1779

556. e Rosen 5th, Chapter 122, p. 1781

557. c Rosen 5th, Chapter 122, p. 1781-1782

558. b Rosen 5th, Chapter 122, p. 1781

A patient suspected of presenting with myxedema coma commonly has a prior history of primary hypothyroidism or previous thyroid surgery. Medication noncompliance or coexisting stressors such as cold exposure, severe infection, or the addition of new medications may precipitate the onset of myxedema coma. By far, infection is the most common cause. The many complex effects of iodine-rich amiodarone on thyroid physiology may lead to asymptomatic abnormalities of thyroid hormone levels, including an elevated TSH, as well as clinically relevant hypofunction of the thyroid gland. Hypothyroidism has been estimated to occur in 1% to 32% of patients taking amiodarone. The most common cause of tertiary adrenal insufficiency and adrenal crisis is functional, from exogenous glucocorticoid administration. Rapid withdrawal of steroids from patients with adrenal atrophy secondary to chronic steroid use may result in collapse and death, especially under circumstances of increased stress. The major mineralocorticoid is aldosterone. The renin-angiotensin system and plasma potassium concentration regulate aldosterone through negative feedback loops. Aldosterone acts to increase sodium reabsorption and potassium excretion, primarily in the distal tubules of the kidneys. The usual laboratory findings in patients with primary adrenal insufficiency include hyponatremia, hyperkalemia, hypoglycemia, and azotemia. Hyponatremia is present in 88% of cases and is usually mild to moderate; severe hyponatremia (200 mg/dl), ketonemia (>1:2 dilutions) and academia (pH